DATA INTERPRETATION REPLICA QUESTIONS THAT HAVE APPEARED IN CAT IN THE LAST 4 YEARS

TABLES 1. It was found that the volume of data transfer in India is the same as that of Singapore. Directions for questions 1 to 3: Answer the following Then which of the following statements is true? questions based on the information given below. (1) Total revenue is the same in both countries. (2) Total revenue in India is about 2 times that of Telecom operators get revenue from transfer of data Singapore. and voice. Average revenue received from transfer of (3) Total revenue in India is about 4 times that of each unit of data is known as ARDT. In the table Singapore. below, the revenue received from data transfer as a (4) Total revenue in Singapore is about 2 times percentage of total revenue received and the ARDT that of India. in US Dollars (USD) are given for various countries. (5) Total revenue in Singapore is about 4 times that of India. Revenue from data ARDT transfer as a 2. It is expected that by 2010, revenue from data Country (in USD) percentage of total transfer as a percentage of total revenue will revenue triple for India and double for Sweden. Assume Japan 13 70 that in 2010, the total revenue in India is twice India 1 9 that of Sweden and that the volume of data Brazil 2 8 transfer is the same in both the countries. What is Thailand 1 11 the percentage increase in ARDT in India if there Israel 4 13 is no change in ARDT in Sweden? Hungary 2 15 (1) 400% (2) 550% Ireland 14 23 (3) 800% (4) 750% Russia 1 18 (5) Cannot be determined

China 2 25 3. If the total revenue received is the same for the Indonesia 2 42 pairs of countries listed in the choices below, Philippines 3 54 choose the pair that has approximately the same UK 13 30 volume of data transfer. Spain 7 14 (1) Philippines and Brazil Sweden 6 18 (2) South Korea and Poland Poland 6 22 (3) Germany and USA Germany 7 24 (4) UK and Spain South Korea 10 24 (5) Isreal and Norway Norway 11 20 USA 11 17 Singapore 9 21

Directions for questions 4 to 6: Answer the following questions based on the information given below.

For admissions to its postgraduate programme, six institutes conducted a common admission test. The test had four sections with the maximum marks in each section being 50. The following tables gives the aggregate as well as sectional cut-off marks fixed by the six institutes. A student will get interview calls only if he/she gets marks higher than or equal to the cut-off marks in each of the sections and his aggregate marks are at least equal to the aggregate cut-off marks as specified by the institute.

Sectional cut-off marks Aggregate cut-off marks Section A Section B Section C Section D Institute 1 41 41 41 172 Institute 2 44 44 171 Institute 3 45 167 Institute 4 42 44 174 Institute 5 44 42 176 Institute 6 40 43 172

4. Arun got calls from all colleges. What could be (1) 0 (2) 17 (3) 21 (4) 31 (5) 37 the minimum aggregate marks obtained by him? (1) 176 (2) 177 (3) 192 6. Chandu did not get a call from even a single (4) 172 (5) 180 college. What could be the maximum aggregate marks obtained by him? 5. Bala got calls from two colleges. What could be (1) 179 (2) 174 (3) 182 the minimum marks obtained by him in a section? (4) 194 (5) 188 1 Directions for questions 7 to 9: Answer the following questions based on the information given below.

There are 100 employees in an organisation across five departments. The following table gives the department- wise distribution of average age, average basic pay and allowances. The gross pay of an employee is the sum of his/her basic pay and allowances.

Number of Average age Average basic Allowances Department Employees (years) pay (Rs) (% of Basic pay) HR 5 46 10000 60 Marketing 3 36 12000 70 Finance 20 31 13000 50 Business Development 35 43 15000 65 Maintenance 10 36 11000 40

There are limited number of employees considered for transfer/promotion across departments. Whenever a person is transferred/promoted from a department of lower average age to a department of higher average age, he/she will get an additional allowance of 10% of basic pay over and above his/her current allowance. There will not be any change in pay structure if a person is transferred/promoted from a department with higher average age to a department with lower average age.

Questions below are independent of each other. Composition Energy drink Carbohydrate Protein Fat Minerals 7. What is the approximate percentage change in (%) (%) (%) (%) the average gross pay of the HR department due A 50 30 10 10 to transfer of a 40 year old person with basic pay of `16000 from the marketing department? B 80 20 0 0 (1) 9% (2) 11% (3) 13% C 10 30 50 10 (4) 15% (5) 17% D 5 50 40 5

8. There was a mutual transfer of an employee E 45 50 0 5 between Marketing and Finance departments and transfer of one employee from Marketing to HR. 10. For the sprinters, he has to prepare a drink As a result, the average age of Finance containing 10% minerals and at least 30% protein. department increased by one year and that of In how many different ways can we prepare this Marketing department remained the same. What drink by mixing at least two ingredients? is the new average age of the HR department? (1) One (2) Two (3) Three (1) 31 (2) 36 (3) 41 (4) Four (5) Five (4) 46 (5) Cannot be determined 11. Which among the following is the combination 9. If two employees (each with a basic pay of having the lowest cost per unit for a drink having `12000) are transferred from Maintenance 10% fat and at least 30% proteins? The drink has department to HR department and one person to be formed by mixing two ingredients together. (with a basic pay of `16000) was transferred from (1) B and C (2) B and E Marketing department to HR department, what (3) B and D (4) C and E will be the percentage change in average basic (5) D and E pay of HR department? (1) 10.5% (2) 12.5% (3) 15% 12. In what proportion should B, C and E be mixed (4) 30% (5) 40% to make a drink having at least 60% carbohydrate at the lowest cost per unit? Directions for questions 10 to 13: Answer the (1) 2 : 1 : 3 (2) 4 : 1 : 2 following questions based on the information given (3) 2 : 1 : 4 (4) 3 : 1 : 2 below. (5) 4 : 1 : 1

An athletics coach was trying to make an energy 13. A drink containing 30% each of carbohydrate and drink which is best suited for the athletes. For this protein, no more than 25% fat and at least 5% purpose he took five of the best known energy drinks minerals is to be made. Which of the following – A, B, C, D and E in the market with the idea of two drinks must be mixed in equal quantities to mixing them to get the result he desired. The table obtain the required drink? below gives the composition of these drinks. The cost (1) A and B (2) D and E (3) B and E of each of these energy drinks per litre is A – 150, (4) C and D (5) A and E B – 50, C – 200 D – 500 and E – 100.

2 Directions for questions 14 to 17: Answer the following questions based on the information given below:

The following table shows the breakup of actual costs incurred by a company in the last five years (year 2004 to year 2008) to produce a particular product.

2004 2005 2006 2007 2008 Volume of production and sale (units) 1500 1350 1650 1800 1800 Costs (Rs) Material 75000 67650 82800 89850 90000 Labour 30000 27000 33150 36225 36000 Consumables 3000 3300 2700 2400 2100 Rent of building 1500 1500 1650 1650 1800 Rates and taxes 600 600 600 600 600 Repair and maintenance expenses 1200 1230 1170 1185 1200 Operating cost of machines 45000 40500 50250 54030 54000 Selling and marketing expenses 8625 8700 8700 8625 8700

The production capacity of the company is 3000 units. The selling price for the year 2008 was `125 per unit. Some costs change almost in direct proportion to the change in the volume of production, while others do not follow any obvious pattern to change with respect to the volume of production and hence are considered fixed. Using the information provided for the year 2008 as the basis for projecting the figures for the year 2009, answer the following questions.

14. What is the approximate cost per unit in rupees, if How many units should the company produce to the company produces and sells 2100 units in the maximize its profit? year 2009? (1) 2100 (2) 2400 (3) 2700 (1) 104 (2) 107 (3) 110 (4) 2850 (5) 3000 (4) 115 (5) 116 17. Given that the company cannot sell more than 15. What is the minimum number of units that the 2550 units, and it will have to reduce the price by company needs to produce and sell to avoid any `5 for all units, if it wants to sell more than 2100 loss? units what is the maximum profit, in rupees, that (1) 470 (2) 525 (3) 576 the company can earn? (4) 1120 (5) 1392 (1) 38100 (2) 36600 (3) 1800 (4) 1900 (5) 2000 16. If the company reduces the price by 5%, it can produce and sell as many units as it desires.

Directions for questions 18 to 21: Answer the following questions based on the information given below.

The table below shows the comparative costs, in US Dollars, of certain items in USA and a select few Asian countries. The models considered are the most popular ones in the respective countries.

Comparative Costs in USA and some Asian countries (in US Dollars) Product USA India Thailand Singapore Malaysia LCD TV 2300 1000 1100 1850 900 Home gym 1900 900 1000 1250 900 Refrigerator 2000 1100 1300 1300 1100 Air conditioner 2300 900 1200 1200 1000 Washing machine 1200 300 450 600 300 Music system 2000 850 1000 1300 800 Digital camera 1600 550 700 900 600

The equivalent of one US Dollar in the local currencies is given below.

I US Dollar equalent India 40.93 Rupees Malaysia 3.51 Ringits Thailand 32.90 Bahts Singapore 1.53 S Dollars

A consulting firm found that the quality of the products was not the same in all the countries above. A poor quality product would result in higher servicing costs over its life time. The cost of poor quality of the products is given in the table

3 Comparative cost of poor quality in USA and Asian Products countries (in US Dollars) USA India Thailand Singapore Malaysia LCD TV 0 300 300 200 400 Home gym 0 500 400 500 500 Refrigerator 0 500 500 400 600 Air Conditioner 0 700 500 500 800 Washing machine 0 500 600 500 400 Music system 0 900 600 400 400 Digital camera 0 500 600 500 600

Note: For all questions assume that the models considered are the ones for which the prices are mentioned.

18. A US citizen requires a refrigerator, air Sl. City of City of Distance between the Fare conditioner and a music system. He can buy No Departure Arrival two cities. (in kms) (Rs). them through an internet portal from any of the 1 A B 280 335 given countries without having to pay for any 2 A C 395 675 transportation costs. Which country will result in 3 A D 425 625 the cheapest overall cost, taking cost of poor 4 A E 625 800 quality into account? 5 A F 672 850 (1) India (2) Thailand 6 A G 675 1225 (3) Malaysia (4) Singapore 7 A H 975 925 (5) USA 8 B C 825 1000

9 B H 875 950 19. Taking the cost of poor quality into account, 10 B I 1050 1225 which country/countries will be the most expensive if one has to buy a music system? 11 B J 1150 1135 (1) India (2) Thailand 12 C D 230 225 (3) Malaysia (4) Singapore 13 C F 205 215 (5) India and Singapore 14 C G 455 550 15 D E 270 295 20. Approximately what difference in Bahts will it 16 D F 312 350 make to a Thai citizen who is touring India if she 17 D G 320 375 were to get a washing machine from India instead 18 D H 475 625 of her native country, taking into account the cost 19 D J 825 1225 of poor quality? One has to pay a duty of 1500 20 E F 625 850 Bahts for transporting the washing machine from 21 E G 485 575 India to Thailand. 22 E H 425 435 (1) 2350 (2) 4050 (3) 5150 23 F G 450 525 (4) 6725 (5) 7500 24 F I 437 475 25 F J 485 575 ` 21. The rupee values increases to 35 for a US 26 G I 255 275 Dollar, and all other things including quality, 27 G J 415 445 remain the same. What is the approximate 28 H I 395 485 difference in cost, in US Dollars, between 29 H J 200 210 Singapore and India for a digital camera, taking 30 I J 230 270 this change into account?

(1) 70 (2) 250 (3) 450 22. What is the lowest price, in rupees, a passenger has (4) 800 (5) No difference to pay for traveling by the shortest route from A to J?

(1) 1135 (2) 1425 (3) 1445 Directions for questions 22 to 26: Answer the (4) 1465 (5) 1670 following questions based on the information given below. 23. The operator plans to introduce a direct service

between cities A and J. The market research An all India tour operator connects ten cities A to J. results indicate that all its existing passengers The table below gives the distance between a pair of traveling between A and J will use the direct service cities and the corresponding price charged by the if it is priced 5% below the minimum price that they operator. Travel is permitted only from the city of pay at present. What should the operator charge, departure to the city of arrival. The customers do not approximately, in rupees, for the direct service? travel by a route when they have to stop at more than (1) 995 (2) 1078 (3) 1349 two intermediate cities. (4) 1372 (5) 1392

4 24. If cities C, D and H are not accessible due to 27. What is the percentage of male employees in the security reasons, what would be the minimum production department? price, in rupees, to be paid by a passenger (1) 40 (2) 45 (3) 50 traveling from A to J? (4) 55 (5) 60 (1) 1140 (2) 1310 (3) 1425 (4) 1475 (5) 1545 28. In the marketing department, twenty five per cent of the post graduates are male. What is the 25. If the prices include a margin of 10% over the difference between the number of female post total cost that the operator incurs, what is the graduates and male employees who are not post minimum cost per kilometer that the operator graduates? incurs for the service from A to J? (1) less than 8 (2) 10 (3) 12 (1) 0.77 (2) 0.88 (3) 0.99 (4) 14 (5) 16 (4) 1.06 (5) 1.08 29. What percentage of employees in the marketing 26. If the prices include a margin of 15% over the department are post graduates? total cost that the company incurs, which among (1) 40 (2) 45 (3) 50 the following is the distance to be covered in (4) 55 (5) 60 traveling from A to J that minimizes the total cost per kilometer for the operator? 30. In the production department, 50% of the males (1) 1085 (2) 1090 (3) 1160 are post graduates. Which of the following (4) 1175 (5) 1195 statements is correct? (1) Except post graduate males, all other groups Directions for questions 27 to 30: Answer the following have the same number of employees. questions based on the information given below: (2) Except males who are not post graduates, all other groups have the same number of The proportion of male employees and the proportion employees. of post graduates in a company are given below. (3) Except post graduates females, all other The company has a total of 800 employees, 80% of groups have the same number of employees. whom are in the production department and the rest (4) Except females who are not post graduates, equally divided between the marketing and the all the other groups have the same number of accounts department. employees. (5) All of the above groups have the same Department Male Post graduates number of employees. Marketing 0.60 Accounts 0.55 0.50 Production 0.55 Total 0.475 0.53

Directions for questions 31 to 33: Buziki motors, a two wheeler manufacturer, introduced a variant in the 125cc category in the beginning of 2009. The number of bookings received in a city for a period 12 months is as given below.

Q1 Q2 Q3 Q4 Month Jan Feb March April May June July Aug Sep Oct Nov Dec Number of bookings 346 412 380 450 308 359 462 333 345 250 506 370 Number of deliveries 200 362 356 445 395 297 496 300 249 420 403

The company promises the delivery of the vehicles within a span of two months i.e., a booking made in January will be delivered in February or March.

The entire period of 12 months is divided into 4 quarters. The price of the motorcycle was increased every quarter. The customer had to pay the total amount at the time of the booking.

Quarter Price (in Rs) per motor cycle Q1 42000 Q2 42500 Q3 43100 Q4 44000

31. In which quarter was the average number of bookings per month, the highest? 33. What is the highest revenue (in `crore) obtained (A) Q1 (B) Q2 (C) Q3 (D) Q4 by the company from the sales of the new 125cc variant in any of the four quarters? 32. The number of deliveries made in December from (A) 4.91 (B) 4.95 (C) 4.99 (D) 5.05 the bookings made in November is how many times the number of deliveries made in August from the bookings made in June? (A) 1.39 (B) 1.58 (C) 2.38 (D) 2.58 5 Directions for questions 34 to 36: The following Percentage of investments in the seven companies table gives the relation between the scaled scores of under the different schemes are as follows: three teams and the number of wins in the matches played. Scheme Scheme Scheme Companies X Y Z Number of Scaled score GB Holdings 5% 20% 15% wins Team A Team B Team C Solar Computers 12% 2.5% 10% 10 – 12 18 19 20 NLP Industries 16% 12.5% 7.5% 13 – 15 22 22 24 Techies’ 2 10% 30% 16 % 16 – 18 28 36 36 Technologies 3 19 – 21 30 27 39 Wiz craft solutions 25% 10% 15% 22 – 24 41 51 46 OBCC bank 15% 10% 22.5% 25 – 27 68 64 63 Live life mutual funds 1 17% 15% 13 % 28 – 30 62 51 52 3 31 – 33 94 99 95 The brokerage firm promises a rate of return of 2% on 34. If the scaled score for two of the teams is the scheme X, 2.5% on Scheme Y and 3% on Scheme Z. same and greater than that of the third team, then which of the following could be the range for the Assume that the rate of return on each of the ‘number of wins’? companies in a scheme is same as the overall rate of (A) 13 – 15 (B) 16 – 18 return of the scheme. (C) > 16 (D) Either (A) or (C) ` 35. For which of the following range for the ‘number 37. If Mahesh invested 12 lakh in the stock brokerage farm, but he immediately withdrew an of wins’ is the difference between the scaled ` scores of team A and team B as a percentage of amount of 3 lakh, then what is the percentage that of team C, the 3rd least? change in the investment in NLP Industries (A) 19 – 21 (B) 10 – 12 because of the withdrawal? (C) 31 – 33 (D) None of these (A) 2% (B) 2.5% (C) 3% (D) 4%

38. Mr. Anil and Ms. Shivani invests an amount of 36. If average scaled score of the three teams is 2.5 ` ` times the average of the corresponding ‘number 7 lakh and 13 lakh respectively. What is the ` of wins’, Then which of the following could be the difference (in ) in their returns on investment range for the ‘number of wins’. made in Wizcraft solutions? (A) 25 – 27 (B) 31 – 33 (A) 250 (B) 300 (C) 22 – 24 (D) All the three (C) 400 (D) More than 500

39. If three persons A, B and C make investments in Directions for questions 37 to 39: A stock the ratio of 10 : 20 : 21, such that their brokerage firm accepts the investments and places investments under the schemes X,Y and Z the amounts invested in seven different companies respectively, then what is the increase in their under three schemes. Scheme X is applicable for an combined return on investment (in `) if the firm investment of 5 to 10 lakh rupees, scheme Y is increases the rate of return on the schemes X, Y applicable for an investment of 11 to 20 lakh rupees and Z by 10%, 20%, and 10% respectively? and scheme Z is applicable for an investment of 21 to (A) 16100 (B) 16300 40 lakh rupees. (C) 17300 (D) None of these

Directions for questions 40 and 41: The following table gives the numbers of music payers, of two companies - NOSY and BOSS, sold in Delhi across three years.

2001 2002 2003 Type of music system NOSY BOSS NOSY BOSS NOSY BOSS Mono speaker 1000 1600 3000 2700 2600 4000 Dual speaker – 1000w 1800 2000 3100 2500 4400 3400 Dual speaker – 2000w 2300 1200 2900 3200 3600 4200 Four speaker – 5000w 1400 2200 3000 4200 4600 3800 Home theatre 1600 2400 3200 2400 4000 5000 Total 8100 9400 15200 15000 19200 20400

40. From 2001 to 2003, for how many types of music points in its contribution to the total sales of the systems is there an increase in the percentage company NOSY? contribution for each of the two companies? (A) Mono speaker (A) 0 (B) 1 (B) Dual speaker – 1000w (C) 2 (D) More than 2 (C) Dual speaker – 2000w (D) Four speaker – 5000w 41. From 2001 to 2003, which type of music system has shown the maximum change in percentage 6 Directions for question 42: The following table 42. Which of the following statement (s) is/are ‘true’? gives the break up of the number of cars sold by Ι. The total number of cars sold by showroom A two showrooms in the first 10 days of their opening. at the end of 7 days lies between 90% and 110% of that sold by showroom B. Day Showroom A Showroom B ΙΙ. In the given period, the total number of cars 1 16 18 sold by showroom A on odd numbered days 2 20 19 is less than 90% of that sold by showroom B 3 35 26 on even numbered days. 4 30 42 (A) Only Ι (B) Only ΙΙ. 5 33 39 (C) Both Ι and ΙΙ (D) Neither Ι nor ΙΙ. 6 24 29 7 51 48 8 63 52 9 60 71 10 79 81

Directions for questions 43 and 44: The following data gives the details of the establishment fee and average number of customers estimated (per year) in franchises of a restaurant chain, Foodies, in class A and class B centers of different cities in India.

Estimated customers Est. fee in class A Est. fee in class B Estimated customers per City per year in class B centers (in ` lakh) centers (in ` lakh) year in class A centers centers Hyderabad 126 75 51,860 42,500 Bengaluru 144 90 60.200 50.246 Pune 132 104 70,000 52,400 Chennai 125 95 48,800 40,000 Kolkata 115 65 45,500 37,000

43. If each customer spends an average amount of 45. Which of the following is definitely false? `240 in restaurants in class A centers and `180 (A) For more than one of the five districts the in class B centers, then in which of the given percentages are continuously increasing or cities will the franchise earn revenues more than decreasing. the establishment fees (for each type of centre), (B) In the given period, the number of dropouts is in one year? the highest in 2006 for at least two districts (A) Hyderabad (B) Pune (C) The total number of dropouts as a (C) Chennai (D) Bengaluru percentage of total number of enrolments, in the given period, is not the highest for Q if it 44. If a person owns two franchises of Foodies in is given that the districts R and S have Pune, one in a class A center and the other in a class B center, and it is estimated that the registered highest percentages in 2002 and average amount each person spends in class A 2006 respectively. and class B centers are `300 and `130 (D) In the given period, the maximum number of respectively, then find the minimum number of dropouts in all the five districts combined was customers, who are required to come to the two registered in 2006, if it is given that R has restaurants together in the first year, such that registered its maximum percentage in 2006 the revenues are not less than the establishment and in any given year each of the fees for each? five districts have equal number of (A) 12.4 lakh (B) 1.24 lakh enrolments in primary schools. (C) 2.4 lakh (D) 9.6 lakh 46. If the percentage of dropouts in a year has Directions for questions 45 and 46: The number of decreased with respect to that in the previous dropouts from primary schools, as a percentage of the total enrollments in a year in five districts across year, than it is considered as an ‘achievement’ for six years are given below. The values represented by a district. What is the minimum number of such ‘–‘ are unknown. ‘achievements’, in the given period, for all the five districts combined? Districts 2001 2002 2003 2004 2005 2006 (A) 5 P 52.3 51.0 45.6 43.9 42.0 40.8 (B) 10 Q 52.4 53.2 54.1 57.3 61.3 61.1 (C) 11 R 45.9 46.2 – 44.5 43.0 – (D) More than 12

S 36.5 – 37.4 – 38.2 39.6 T 41.2 43.4 42.6 44.5 44.1 45.0 7 Directions for questions 47 and 48: The following 51. Traffic flowing from which of the given cities is the table gives the temperatures in six cities at three maximum? different times of the day. (A) P (B) Q (C) T (D) U

City 5.00 a.m. 12 noon 6.00 p.m. 52. What is the least difference between the traffic P 24 42 29 flowing from a particular city and traffic flowing to Q 25 46 28 the same city? R 25 44 32 (A) 120 (B) 158 (C) 98 (D) 232 S 32 49 31 T 30 46 30 Directions for questions 53 and 54: The following U 22 44 26 table gives the different payments options for a ` 1 lakh loan provided by a rural bank to persons with 47. If the increase in temperature from 5 a.m. to different income levels, depending on the time in 12 noon is linear, then what is the temperature in which they would repay the loan. city Q at 10 a.m.? (A) 30°C (B) 32°C (C) 36°C (D) 40°C Payment options Income level (interest amount in `) (in ‘000 Rs) 48. If the decrease in temperature from 12 noon to 1 year 2 years 3 years 6.00 p.m. is linear, then in which city was the 20 – 40 2,000 3,000 4,000 temperature the highest at 3.30 p.m.? (A) Q (B) R (C) S (D) U 41 – 60 2200 3,600 4,100 61 – 80 2,600 3,800 4,400 Directions for questions 49 to 52: The following table gives the details about the traffic flow on a 81 – 100 2,900 4,000 4,800 particular day through the roads connecting six different cities. For example, 846 vehicles travel A person with an income between `41,000 to from P to Q and 964 travel from Q to P. So, total `60,000 has to repay the loan of `1 lakh with an traffic on the road connecting the cities A and B is interest amount of `2,200 if the loan is repaid in 846 + 964 = 1810. Assume that these are the only 1 year, `3,600 if the loan repaid in 2 years and so on. cities interconnected and traffic flow is among these The interest amount to be paid gets multiplied by the cities only. factor of loan amount (for example a loan of `3.4 lakh lent to a person with an income between `20,000 to P Q R S T U `40,000, in one year, accumulates to an amount (in `) of P – 846 808 400 472 820 3,40,000 + 3.4 × 2,000 = 3,46,800.) Q 964 – 564 540 840 844 R 536 664 – 248 888 200 53. Mr. A, whose annual income is `45,000 takes a S 848 242 624 – 478 428 loan of `2.2 lakh and promises to repay it in T 484 364 784 672 – 648 2 years. Mr. B, whose annual income is `76,000 U 672 496 528 992 372 – takes a loan of `3.6 lakh and promises to repay it in 2 years. What is the difference in the interest 49. The maximum traffic flow occurs on the road amounts (in `) they have to pay? connecting which two cities? (A) 5,440 (B) 5,760 (C) 6,120 (D) 6.260 (A) P – Q (B) P – R (C) T – R (D) None of these 54. A person with an annual income of `96,000 takes two loans-one of `5.6 lakhs for 3 years and 50. The total number of vehicles passing through the another of `6.4 lakh for 2 years. What is the road connecting any two of the given cities is the average of the interest amount paid by him? second least for (A) `24,840 (B) `26,000 (A) S – U (B) R – S (C) `26,240 (D) `28,140 (C) P – T (D) None of these

Directions for questions 55 and 56: The following table gives the distribution of number of male employees and female employees owning a four wheeler or a two wheeler or both in three companies X, Y and Z.

Company Four Wheeler Two wheeler Neither Male employees 45% 65% 5% X Female employees 30% 70% 10% Male employees 50% 60% 20% Y Female employees 25% 80% 15% Male employees 36% 54% 20% Z Female employees 24% 67% 9%

The number of male employees in company X is 70% of the total number of employees in the company and the number of female employees in company Y and Z is 40% each of the total number of employees of their respective companies.

8 55. In which company is the percentage of The table below shows the revenue of the company employees who own both two wheeler and four from each of the five models in both the years. wheeler, the highest? (A) X (B) Y Revenue Revenue (C) Z (D) Cannot be determined Model (in ` crore) (in ` crore) In 2007 In 2008 56. What percentage of the male employees in RL-100 78 87.75 companies Y and Z together own either a four BCZ 93.75 105 wheeler or a two wheeler but not both, if it is given that the total number of employees in both Thunder 93 105 the companies is the same? WB-150 90 103.5 (A) 40% (B) 50% Muzzle (C) 80% (D) None of these 93.6 99

Directions for questions 57 and 58: The following 57. For which model is the percentage increase in table gives the details about the percentage the average selling price per bike, the highest? distribution of the total bikes sold by ACE Motors Ltd. (A) RL-100 (B) Thunder The percentage distribution was the same in 2007 (C) WB-150 (D) Muzzle. and 2008. The total number of bikes sold in 2007 is 1,50,000 which is the same as that in 2008. 58. What is the average of the selling prices (in `) of the five models in 2007? Model Percentage of total bikes sold (A) 37,600 (B) 33,600 RL-100 13% (C) 32,400 (D) None of these. BCZ 25% Thunder 20% WB-150 30% Muzzle 12%

Directions for questions 59 and 60: The following table gives the performance of four companies, all listed on National Stock Exchange (NSE), from 2001 to 2010.

Company Ι Company ΙΙ Company ΙΙΙ Company ΙV Share price Dividend Share price Dividend Share price Dividend Share price Dividend Year (in `) (in `) (in `) (in `) (in `) (in `) (in `) (in `) 2001 128 120 283 112 148 128 400 128 2002 132 116 289 115 152 132 420 124 2003 126 123 295 128 163 145 432 136 2004 148 152 296 138 168 148 440 144 2005 158 148 312 142 172 140 453 148 2006 123 121 328 154 184 152 448 132 2007 172 113 324 132 196 128 432 120 2008 164 109 345 106 212 136 464 112 2009 128 105 360 121 252 138 484 128 2010 132 102 364 143 286 140 496 132

For any company,  + −   Px Px +1 2Px −1  Gx = Dx +    2 

Where Gx is gain an company;s share in yea x. Dx is the divided declared by the company in year x. Px and Px + 1 are share prices of the company in year x and x + c respectively.

59. What was the gain from the shares of company 60. What was the highest percentage increase in the IV in 2006? gain from the shares of company III in a year with (1) 116 (2) 132 (3) 148 (4) 138 respect to the previous year? (1) 13.48% (2) 29.38% (3) 16.32% (4) 38.45%

9 61. The following table gives the per capita income of What percent of the coal production of the top countries in the year 2004. From the table four (i.e, the four highest) countries is the coal determine the number of countries having their production of the bottom four (i.e. the four lowest) per capita income more than 40% of the median countries? per capita income of these countries? (1) 10.66% (2) 8.98% (1) 9 (2) 10 (3) 11 (4) 8 (3) 11.32% (4) 12.45%

Per capita income (gross) in US $ Directions for questions 64 and 65: The following Switzerland 24,369 table gives the gender, height, weight and age of New Zealand 15,350 fifteen students of a college who have cleared the Lithuania 4,965 preliminary round of the selection process for being Romania 2,916 selected in the Air Force. The names of the students Spain 11,692 being denoted by A, B, C, D, . . . . . and O. Sweden 13,746 United States 23,484 Height Weight Age Names Gender France 13,477 (in cm) (in kg) (in years) Mexico 3,523 1 A M 164 64 22 Hong Kong 10,372 2 B M 169 62 21 United Kingdom 19,207 3 C F 152 49 17 Brazil 5,663 4 D M 148 68 18 Germany 24,337 5 E F 154 78 21 6 F M 172 68 23 Directions for question 62: Bolvo bus service has 7 G M 168 67 27 ` the following revenue data (in crore) regarding its 8 H M 165 70 22 operations in 2007. 9 I F 145 58 20 Non A/c 10 J M 152 78 18 A/c A/c semi Non A/C semi- Total 11 K M 156 64 18 sleeper sleeper general sleeper 12 L F 146 51 19 Inter-state 13 M F 138 56 23

services 14 N M 171 67 24 Intra-state 15 O F 162 60 25 240 2880 services Total 1200 Air Force follows a selection criterion where the height and weight of a person must lie in one of the A/c sleeper and A/C semi-sleeper accounted for following ranges: 37.5% of the total revenue whereas non A/c semi- sleeper accounted for 25% of the total revenue. 50% Male of Intra-state services revenue was generated from non A/c general services. HEIGHT (in cm) WEIGHT (in kg) 60% of the total revenue generated was from Intra- state services. The revenue generated from A/c 155 – 160 58 – 62 sleeper in inter-state services to that in intra-state 160 – 165 60–64 services was in the ratio of 1 : 2 165 – 170 63–68

62. What was the total revenue (in ` crore) generated 170 – 175 66–74 from non A/c general in Intra-state services? (1) 600 (2) 1440 (3) 950 (4) 600 Candidates whose height is less than 155 cm or more than 175 cm, are not eligible for selection. Directions for question 63: Female 63. The following table gives list of the major coal producing countries in the world along with the WEIGHT (in HEIGHT (in cm) total production of coal in the year 2007(in million kg) tonnes) 145 – 150 48 – 52 Country Coal production 150 –155 52 –56 China 2536.7 155– 160 55 –59 Russia 314.2 160 – 165 58 – 62 Poland 145.8 Ukraine 76.3 Candidates with weight less than 145 cm or more India 478.2 than 165 cm are not eligible for selection. Turkey 76.6

United States 1039.2 64. Find the ratio of the number of male and female Australia 393.9 students who were eligible for selection. Germany 201.9 (1) 2 : 1 (2) 5 : 2 (3) 3 : 2 (4) 4 : 1 Indonesia 174.9 10 65. If x represents the number of male students Directions for questions 70 to 72: The following whose age lies in the range of 18 to 22 years table gives the exports and imports (values given in (both inclusive) and y represents the number of millions of `) of the 4 companies in the years 2002 – females students whose weight lies in the range 03, 2003 – 04, 2004 – 05 of 49 to 58 kg (both inclusive) then (1) x > 2y (2) x < y Total (3) x = y (4) 2x = 3y Company Year Exports Imports trade

Directions for questions 66 and 67: The following 2002 - 03 7.13 5.14 12.27 Rahul table gives the cost and revenue of a company for a 2003 - 04 12.15 11.67 23.82 & co period of five years from 2005 to 2009. The total cost 2004 - 05 15.3 17.41 32.71 is the sum of the costs under three heads H1, H2 and 2002 - 03 12.4 11.61 24.01 Chandu H3. Operating Expense of the company for each year 2003 - 04 14.1 16.31 30.31 & co is equal to 20% (H1) + 25% (H2) + 30% (H3) the 2004 - 05 16.2 17.1 33.3 profitability of the company in each year is defined as 2002 - 03 5.4 8.72 14.12 Shiva Operating Expense 2003 - 04 9.3 9.39 18.69 . Study the given table carefully & co Re venue 2004 - 05 12.1 13.17 25.27 and answer the questions that follow: 2002 - 03 6.54 7.46 14.00 Kanta All values are given in '000s of `. 2003 - 04 10.41 11.51 21.92 & co 2004 - 05 13.73 14.33 28.06

Year H1 H2 H3 Revenue 2009 20.8 30.6 40.8 104.2 70. Which of the following company registered the highest percentage growth in exports from 2003 – 2008 21.2 24.3 38.2 96.6 04 to 2004 – 05? 2007 29.6 38.4 21.6 112.4 (1) Rahul & co. (2) Chandu & co. 2006 30.8 23.4 42.4 128.2 (3) Shiva & co. (4) Kanta & co. 2005 24.8 42.8 36.4 130.6 71. Which company registered the least growth rate 66. In which of the following years was the in imports from 2002 – 03 to 2003 – 04? profitability of the company the least? (1) Rahul & co. (2) Chandu & co. (1) 2005 (2) 2006 (3) Shiva & co. (4) Kanta & co. (3) 2007 (4) 2008 72. Which company had the highest trade-deficit (= 67. What was the maximum percentage decrease in imports – exports) in 2004-05? the total cost of the company in a given year with (1) Rahul & co. (2) Chandu & co. respect to the previous year? (3) Shiva & co. (4) Kanta & co.

(1) 7.04% (2) 7.24% Directions for questions 73 and 74: Five (3) 7.11% (4) 8.92% companies held examinations for all the employees who are in their probation period. Directions for questions 68 and 69: The following table gives the distribution of the number of students The following table gives the details of all the based on the marks obtained in a certain examination employees who have taken the exam for five sections A, B, C, D and E. Students were categorised as per their marks being 'less than 45', No. of % of employees from 45 to 85 and greater than 85. employees No. of employees who got more Company who who wrote the than 90% of M is the marks obtained. crossed exam marks the cut off Sections M < 45 45 < M ≤ 85 M > 85 A 180 10 300 B 225 8 450 A 28 72 24 C 150 12 250 B 15 68 36 D 400 16 600 C 18 52 28 E 300 20 575 D 29 58 47 E 30 60 35 73. If the employees who do not clear the cut offs are rejected, then which company rejected the 68. What percentage of the total number of students maximum number of employees? got scores less than 45? (1) B (2) C (3) D (4) E (1) 15.6% (2) 18.8% (3) 21.2% (4) 20% 74. What is percentage of employees who got more 69. If the qualifying mark in the paper is 48, then the than 90% of marks out of the total number of maximum number of students from a section who employees who cleared the cut off for all passed in that examination was companies combined (approximately)? (1) 96 (2) 104 (3) 105 (4) 95 (1) 20% (2) 22% (3) 24% (4) 28% 11 Directions for questions 75 and 76: The following 2009 P 7 11 485 table gives the sales of 5 companies in 2008 and Q 4 15 690 2009 R 8 10 775 S 6 9 1245 No. of units T 9 12 865 Price/unit Closing 2008 Company products (in `) stock (in 1000) 75. Which company had the least sales in the year 2009? P 8 12 500 (1) P (2) S (3) Q (4) T Q 6 14 750 76. In which year did R have lower sales? R 5 9 675 (1) 2008 S 10 11 890 (2) 2009 T 11 13 1200 (3) Both 2008 and 2009 (4) None of these

Directions for questions 77 and78: The following table gives the number of members in seven families and the details regarding their income and expenses.

Average income of the Expenses of the Overhead No. of members family in (`) family in (`) expenses in (`)

Kapoor family 6 24500 9000 3000 Khanna family 5 21000 11000 2500 Kirsten family 7 24000 14000 3750 Kumble family 4 35000 12500 4250 Khan family 6 27500 13000 6000 Kittu family 3 40000 14200 3250 Kala family 7 28000 17000 4375

77. What is the total savings made by all the families 78. If the average income of the Khan family (in `)? increased by 2% where as the expenses of the (1) 913175 (2) 923175 family decreased by 3%, then the savings of the (3) 923075 (4) 921075 Khan family would increase by (1) `3370 (2) `3570 (3) `3870 (4) `3670

Directions for question 79: The following table gives the diet statistics of 10 Drinks named P to Y. The values are in percentages.

Drinks Proteins Vitamins Carbohydrat Fats Sugar content P 16% 24% 12% 27% 21% Q 21% 18% 18% 14% 29% R 17% 25% 20% 24% 14% S 23% 26% 20% 16% 15% T 18% 29% 19% 18% 16% U 25% 21% 16% 15% 23% V 24% 22% 14% 19% 21% W 16% 29% 15% 17% 23% X 27% 24% 14% 18% 17% Y 19% 25% 18% 14% 24%

A healthy drink is considered to have at-least 20% of proteins, at-least 23% of vitamins and at-most 20% of other ingredients. Otherwise, it is considered as an unhealthy drink.

79. What is the ratio of healthy drinks to unhealthy drinks in the given group? (1) 2 : 3 (2) 3 : 7 (3) 1 : 4 (4) 1 : 9

12 Directions for questions 80 and 81: The following Directions for questions 82 and 83: The following table shows the number of students in government table gives the number of students in different schools in six different states of India during 2007, sections A, B, C, D, E and F of a school in 2007 2008 and 2009.

Students in Government schools (in ‘000) Section Students New students

A 18 12 State 2007 2008 2009 Andhra Pradesh 15.4 17.2 16.1 B 12 4 M.P 21.2 19.6 20.9 C 20 8 U.P 20.1 21.4 22.1 D 17 10 Karnataka 18.7 17.3 19.6 E 14 11 Kerala 16.3 18.5 17.9 F 19 9 Tamil Nadu 14 19.2 20.3

80. In 2008, which state experienced the maximum 82. With respect to the number of students in each increase in the number of students studying in section, how many sections have the number of government schools with respect to that in the students more than the median of the number of previous year? students in a section for the given sections? (1) Andhra Pradesh (2) U.P (1) 2 (2) 3 (3) 4 (4) 1 (3) Kerala (4) Tamil Nadu 83. The student who fails in any class is retained in the 81. Which state has shown a consistent increase in same class and section. Which of the following the number of students studying in government sections have the highest number of failed schools from 2007 to 2009? students? (1) M.P (2) U.P (1) A (2) B (3) C (4) D (3) Kerala (4) Karnataka

Directions for questions 84 and 85: The following table gives the details of the area utilized and the production of wheat by 3 countries P, Q and R from 2003 to 2008.

P Q R Area Production Area Production Area Production 2003 2.1 5280 1.7 2850 3.5 5450 2004 2.6 1380 2.2 4850 2.7 4250 2005 1.8 2790 1.5 3950 2.6 4900 2006 1.9 5550 2.4 4800 2.3 4650 2007 2.3 5950 2.5 6800 2.1 3350 2008 3.4 5180 1.9 7800 3.2 4880

Pr oduction Yield return = Unit area

84. Which of the following is the highest yield return 85. In which year is the percentage increase in the obtained by country R? yield return the highest for country Q? (1) 2021 (2) 1885 (3) 2011 (4) 1914 (1) 2005 (2) 2006 (3) 2007 (4) 2008

Directions for questions 86 and 87: The following graph shows the average marks obtained by the students and the number of students. 100

90 • • 80

• • • • 70

60 • • • • • • • 50 40

Marks Average • • • • • • • 30 20 • • • • • • • • 10

50 100 150 200 250 300 350 400 450 Number of Students (in hundreds) 13 The table given below shows the average marks 88. In which state are the number of students the obtained and the number of students in each class. third highest this year? (1) Assam Average Number of students Class (2) MP marks (in hundreds) (3) Orissa I 45 450 (4) AP II 60 325

III 31 120 IV 17 180 Directions for question 89: Select the correct V 57 220 alternative from the given choices. VI 37 110 VII 83 180 89. Lakshmi, an employee, wants to invest in 3 types VIII 71 240 of business X, Y and Z. The following table gives IX 62 305 the investment and revenue obtained by Lakshmi X 79 400 from her investments

86. The statement "the higher the average marks, the higher the number of students" is true in which of Name of the the following classes? X Y Z (1) II, VIII, X (2) III, V, VII busine (3) II, IX, X (4) I, IV, VI ss Investments 87. The statement "the lower the number of the (in 16.2 14.5 12.9 students higher is the average marks" is true for lakhs) which of the following classes? Revenue (in 21.2 18.4 16.5 (1) VII, VIII, V (2) I, X, VI lakhs) (3) I, II, V (4) V, VII, IX

Directions for question 88: The following table Which of the following would be the most gives the details of the number of students in 6 states profitable investment for Lakshmi if she spends 20% of the revenue earned from each business No.of students Change in this year to maintain her house? (in lakhs) (in '000) (1) X AP 13 +21 (2) Y UP 17 +46 (3) Z − MP 16 210 (4) Both (1) and (3) Bihar 18 +114 Assam 19 −612 Orissa 12 −112

Directions for questions 90 and 91: The table given below gives the details of income and expenditure for some states in different regions in 2006 and 2007

Per capita income Per capita expenditure Region Income (in ` crore) Expenditure (in ` crore) (in ` crore) (in ` crore)

2006 2007 2006 2007 2006 2007 2006 2007

North 16.2 17.3 18.1 17.6 250 265 215 221 J & K

Punjab 15.1 16.5 15.5 16.9 261 272 218 224

South 18.3 17.5 18.9 17.9 271 281 224 236 AP Karnataka 14.6 16.3 15.1 17.3 283 294 241 249 East 17.5 18.4 17.8 18.9 241 261 213 232 West Bengal Assam 12.1 13.5 12.7 13.8 264 270 222 234 West 19.4 20.2 19.8 20.9 256 271 233 241 Gujarat Maharashtra 19.6 19.9 19.9 20.4 245 259 219 232

90. Which of the following regions had the ratio of income to expenditure in 2007 the highest? (1) North (2) South (3) East (4) West 14 91. What is ratio of the number of states in which per capita income increased by more than 5% to the number of states in which per capita income did not increase by more than 5%? (1) 1 : 7 (2) 1 : 1 (3) 3 : 5 (4) 1 : 3

BAR GRAPH

Directions for questions 1 to 4: Answer the following questions based on the information given below.

The bar chart below shows the revenue, in million US Dollars(USD), of a cosmetics company. The data covers the period 2003 to 2007 for the United States(US) and Asia. The bar chart also shows the estimated revenues of the company for the period 2008 to 2010.

500

D 450 400 350 300

250 200 150 100

Revenue in million US 50 0

2003 2004 2005 2006 2007 2008 2009 2010 Year US Asia

1. The difference between the estimated revenue in 3. Consider the annual percent change in the gap Asia in 2008 and what it would have been if it between revenues in the US and Asia. What is were computed using the percentage growth rate the year in which the absolute value of this of 2007(over 2006) is closest to change is the highest? (1) 25 (2) 40 (3) 10 (4) 5 (5) 0 (1) 30-04 (2) 05-06 (3) 06-07 (4) 08-09 (5) 09-10 2. In 2003, sixty percent of the people who used the company’s products in Asia were men. Given that 4. While the revenues from Asia has been growing women who used the company’s products steadly towards that of the US, the growth rate in increase at the rate of 10 percent per annum and Asia seems to be declining. Which of the men at the rate of 5 percent per annum, what is following is closest to the percent change in the approximate percentage growth of customers growth of 2007 (over 2006) relative to the growth between 2003 and 2010 in Asia? The prices of rate of 2005 (over 2004)? the company’s products are volatile and may (1) 17 (2) 10 (3) 35 (4) 60 (5) 100

change each year. (1) 62 (2) 15 (3) 78 (4) 84 (5) 50

Directions for questions 5 and 6: The first part of the bar graph gives the number of animals of a given species in the Amazon forest as a percentage of the total number of animals of that species in South America and the second part of the bar graph gives the number of animals of that species in South America as a percentage of the total number of animals of that species in the world. The total number of deers and wild bisons in the world are 24,000 and 18,000 respectively. The total number of animals of the five given species in South America is 25,800. 95% 80% 80% 75% 70%

42% 40% 25% 30% 27%

Pythons Deers Wild bisons Wolves Bears

15 5. If the number of Pythons and Bears in South 6. Using the information in the previous question, America are 4800 and 4200 respectively, then arrange the animals in the decreasing order of what is the number of Wolves in the Amazon their number in the Amazon Forest? forest? (A) Deers, Wild bisons, Deers, Wolves, Bears, (A) 3,990 (B) 4,050 Pythons. (C) 4,200 (D) 4,320 (B) Wild bison, Deers, Wolves, Bears, Pythons. (C) Deers, Wolves, Wild bisons, Bears, Pythons. (D) Deers, Wolves, Wild bisons, Pythons, Bears.

Directions for questions 7 to 9: The following graphs shows the sales (by volume) of four PC manufacturing companies P, Q, R and S, in Hyderabad across four years.

6000 5800 5600 5400 5400 5000

4850 5000 4800 4500 4500 4480

4350 4200 4200 4000 3500

3000 3000 2800

2000

1000

0

2006 2007 2008 2009

P Q R S

7. The sales volume of which company increased (A) 6.61 (B) 6.73 (C) 6.95 (D) 7.31 by the highest percentage from 2006 to 2008? (A) P (B) Q (C) R (D) S 9. The market share of which of the following is the highest if these are the only companies in the 8. If in 2010, company S goes bankrupt and the market and price per PC of the companies P, Q, sales volume of the other companies increases R and S in each of the given years was in the by 10% each, when compared to that in 2009, ratio 1 : 2 : 1 : 2? then what is increase, in percentage points, in the (A) S in 2006 (B) R in 2008 market share of company Q if these are the only (C) S in 2007 (D) S in 2009 companies in the market and the cost of PC is same for all the companies?

Directions for question 10: The following table gives the distribution of the number of handsets sold by a mobile- phone manufacturing company from 2004 to 2006. P, Q, R and S denote the different models of mobile-phones sold every year. Further it is given that the selling prices of the four models of mobile phones in the year 2004,SPP, SPQ, SPR, SPS were in the ratio 3 : 4 : 5 : 6 and the ratio of the selling price of each model in the year 2004, 2005 and 2006 was 2 : 3 : 4.

3000

2500 S

2000 S S R

1500 R R Q Q 1000 Q 500 P P

0 P 2004 2005 2006

10. The sales revenue of R in 2006 was more than the sales revenue of Q in 2004 by what %? 2 (1) 50% (2) 56.25% (3) 66 % (4) 75% 3 16 Directions for questions 11 to 13: The following bar graphs gives the sales by volume of 3 cars sold in the market during 2007 to 2009 and also the number of units of these cars produced in these years.

25 22 20 20 20 18 18 16

15 10 9 10 8

5

Sales by volume (in '000)

0 Alto Swift Estilo

2007 2008 2009

30

25 25 22 22 21 21 20 20

15 14 13 15

10

'000) (in Production 5

0 Alto Swift Estilo 2007 2008 2009

11. In which year is the ratio of the total production to 13. Production − sales = Exports the total sales of all 3 cars the highest? In which year is the ratio of the exports to sales of (1) 2007 (2) 2008 Swift the highest? (3) 2009 (4) Both (1) and (2) (1) 2007 (2) 2008 (3) 2009 (4) Both (1) & (2) 12 In which year is the ratio of the production of Alto to the sales, the highest? (1) 2007 (2) 2008 (3) 2009 (4) Both (1) and (2)

PIE CHART

Directions for questions 1 to 6: The total scaled scores obtained by a student in five AIMCATs is as shown in the table below. An AIMCAT has five sections – Quantitative Ability (QA), Logical Reasoning (LR), Verbal Ability (VA), Reading Comprehension (RC) and Data Interpretation (DI).

Exam Total scaled scores AIMCAT 1 300 AIMCAT 2 280 AIMCAT 3 360 AIMCAT 4 320 AIMCAT 5 350

17 The following pie charts give the distribution of the scaled scores in each AIMCAT.

AIMCAT 1 AIMCAT 2

RC VA DI DI 20% 25% 19% 20%

LR QA 11% 17% RC 15% QA VA

37.5% 12.5% LR 23%

AIMCAT 3 AIMCAT 4

QA DI DI 25% 22.5% 25% QA 1 33 /3%

RC 15% VA 10% RC VA LR 20% 162/ % 3 12.5% LR 20% AIMCAT 5 DI

QA 18% 24%

RC 20%

VA LR 26% 12% NOTE: The scores given are actually the ‘scaled scores’. The ‘actual scores’ are obtained by dividing the scaled scores by the ‘scoring factors.’ The maximum scaled scores in the five sections are also given below

Section QA LR VA RC DI Scoring factor 1.5 1.25 2.5 1.25 1.2 Maximum scaled score 150 100 125 75 120

1. The ‘scaled score’ obtained by the student in the ‘significant’. In how many of the five AIMCATs, VA section in AIMCAT 3 is what percent of the the student shows a ‘significant’ performance? maximum possible’ actual score’ in that section? (A) 1 (B) 2 (C) 3 (D) 5 (A) 48% (B) 60% (C) 80% (D) None of these 4. From AIMCAT 1 to AIMCAT 5, in which section did the student have the highest percentage 2. What is the least difference between the ‘scaled increase in the scaled score? score’ in the RC section and the maximum (A) LR (B) QA (C) RC (D) DI

possible ‘actual score’ in that section in any of the 5. From AIMCAT 1 to AIMCAT 5, in which section, given AIMCATs? did the student have the least percentage change (A) 4 (B) 6 (C) 11 (D) 19 in the actual score?

(A) LR (B) VA (C) RC (D) DI 3. In an AIMCAT, if in at least three of the five sections a candidate has a ‘scaled score’ in a 6. In which AIMCAT was the marks obtained by the section greater than 80% of the maximum student in the RC section the highest? possible ‘actual score’ in that section, the (A) AIMCAT 4 (B) AIMCAT 5 performance of the candidate is considered to be (C) AIMCAT 3 (D) AIMCAT 1 18 Direction for question 7: 1000 students in a school Painting have to choose one extra-curricular activity (in which 15% they are interested) among 5 activities – Dancing, Dancing Singing, Printing, Embroidery classes and Karate. Karate 45% Only boys chose Karate, and only girls chose 15% Embroidery classes. The ratio of the number of boys to girls in painting is 1 : 1. 80% of the students who choose Dancing are boys and 80% of the students who choose singing are girls. The following pie chart Singing gives the distribution of the students in the five 20% activities. Embroidery 5% 7. If the students who selected Painting and Singing are made to sit in the same class then what would be the ratio of boys to girls in that class? (1) 23 : 45 (2) 1 : 2 (3) 23 : 47 (4) 23 : 49

Direction for question 8: The following pie charts give the number of employees in central government jobs and state government jobs in 6 states of India

MP AP MP 14% 20% 8% AP UP UP 25% 7% 12% TN Karnataka Karnataka 25% TN 9% 20% 20%

Kerala, Kerala, 25% 15% State Govt. Jobs Central Govt. Jobs

8. The number of employees in central government jobs and in state government jobs are in the ratio 6 : 1. Find the ratio of the number of employees in central govt. jobs in AP to that in state jobs in Kerala? (1) 6 : 1 (2) 4 : 1 (3) 5 : 2 (4) 3 : 1

Directions for questions 9 and 10: The following pie charts give the details of all the employees of 2 companies P & Q: HR dept Academic dept 10% HR dept 17% 21%

Operations

dept Academic 30% dept 60%

Operations dept

62%

Total number of employees in P = 17500 Total number of employees in Q = 18000

9. What percent of the employees in both the 10. What is the approximate ratio of the number of companies belong to the HR department? employees in the Academic department to the (1) 17.5% (2) 15.5% (3) 16.1% (4) 17% Management department (HR dept + operation dept) in both the companies combined? (1) 0.71 (2) 0.62 (3) 0.9 (4) 0.84

19 Pie Charts + Bar Charts

Directions for questions 1 and 2: In a management institute, students opt for various disciplines. The distribution of students across disciplines is shown in the pie chart and the ratio of the number of males and females in each discipline is shown in the bar chart. The institute has sixteen centres in the country and the students whose data is represented in the following charts are from those centres in the year 2009-2010.

Study the given charts carefully to answer the following questions.

Total number of students = 10080. HR 25% Mar keting 37.50%

systems 15%

Finance operations 12.50% 10%

15

10

5

0 Marketing HR Finance Operation Systems

Males Fe m a le s

1. In the year 2009 – 2010, the total number of 2. For which discipline was the difference between female students in the institute was less than the the number of male and female students the total number of male students by what highest? percentage? (1) H R (2) Finance (1) 11% (2) 9.2% (3) 10.6% (4) 12.4% (3) Marketing (4) Systems

20 LINE GRAPH

Directions for questions 1 and 2: The following graph gives the relation between the total expenses and the number of units produced in a factory.

10000

8000

6000

4000

2000

Total expenses (in Rs.) 0 50 100 150 200 250 300 350 400

Number of units produced

Assume that all units produced are sold and that on a normal day 200 units are produced. The selling price of each unit is `35.

1. On a particular day if 300 units are sold, find the 2 → Geyser + Refrigerator percentage change in profit when compared to a 3 → Geyser + Refrigerator + TV normal day 4 → Geyser + Refrigerator + TV + Washing machine (A) 100 (B) 150 (C) 200 (D) 250 5 → Geyser + Refrigerator + TV + Washing machine + Grinder ` 2. What is the average additional cost (in ) per unit produced in comparison to a normal day, when 3. Which of the following is true? 350 units are produced? (1) The energy consumed by TV for 3 days is (A) 18 (B) 23 (C) 25 (D) 28 more than that of Refrigerator for 3 days. (2) The energy consumed by Geyser for 4 days Directions for questions 3 and 4: A family uses the is less than that of Grinder for 7 days. following electrical appliances– TV, Refrigerator, (3) The energy consumed by Washing machine Geyser, Washing machine and Grinder. in a week is less than that of Geyser for 2 weeks. The monthly electricity bill generated has two (4) The energy consumed by TV for 2 days is components- a fixed cost of `60 and a variable cost less than that of Washing machine for a of `0.35 per kWh. week.

The family uses Refrigerator throughout the day, 4. If the fixed cost increases by 25%, then what Geyser for 2 hours, Washing machine for 0.5 hours, would be the percentage increase in the total cost Grinder 0.25 hours and watches TV for 15 hours of energy consumption by the family in a month everyday. of 30 days. (1) 10% (2) 12.5% The line graph given below shows the energy (3) 15% (4) 19% consumption of the above mentioned appliances in a week for the family. DATA SUFFICIENCY

Directions for questions 1 to 4: Each question is 50 followed by two statements, A and B. Answer each 40 question using the following instructions: 40

39 30 35 Mark (1) If the question can be answered by using the statement A alone but not using the

20 statement B alone 21 Mark (2) If the question can be answered by using

10 the statement B alone but not by using

7 the statement A alone. 0 Mark (3) If the question can be answered by using either of the statements alone. Energy Consumption (in KWh) 012345 Mark (4) If the question can be answered by using both the statements together but not by 1 → Geyser either of the statements alone. 21 Mark (5) If the question cannot be answered on Numbers the basis of the two statements. Directions for questions 1 and 2: 1. In a particular company, sixty employees were managers. Ten among them were also among Mark option the people who had newly joined. How many (1), if the question can be answered by any one of employees in the company were newly joined? the statements alone but not by the other. (A) Sixty percent of the newly joined employees (2), if the question can be answered by either were not managers. statement alone. (B) All the newly joined employees were not (3), if the question can be answered by combining necessarily managers. both the statements but not by each statement alone. 2. Five people Amar, Babu, Craig, David and (4), if the question cannot be answered even after Edward were the only ones who participated in a combining both the statements. chess tournament. They were ranked on the basis of the points they scored. David got a 1. Which of x, y, z is the maximum? higher rank as compared to Edward while Babu A. xy = 18 and yz = 21 got a higher rank as compared to Craig. B. xz = 42, where x, y and z are natural Craig’s rank was lower than the median. numbers. Who among the five got the highest rank? (A) Amar got the last rank. 2. If ONE = O + N + E, i.e. considering 'O'=6, N= 7 (B) Babu was not among the top two rankers. and E = 8 we will get ONE = O + N + E = 6 + 7 + 8 = 21. 3. Thirty percent of the students of a school are Find the value of SEEN, if SEVEN = 19 and all boys. Ten percent of the girls in the school are alphabets have distinct values which are natural athletes. What is the percentage of boys in the numbers. school who are athletes? A. FIVE = 14 (A) Twenty five percent of the students are athletes. B. NINE = 7 (B) Number of boys in the school who are athletes is 20% more than the number of girls Directions for question 3: This question is based on who are athletes. the following information.

4. In a basketball match, team A was trailing by Raju's dad goes to various temples on certain days in 25 points at the end of the first half. Did it win the a year of 365 days which are numbered 1, 2, ….365. match? He goes to the Shiva temple on days which are (A) In the second half team A scored 35 points. (B) The opponent scored 35 points in the match. multiples of 3. He goes to the Venkateshwara temple on days which PPL are multiples of 4. He goes to the Saibaba temple on the days which are Directions for question 1: multiples of 7.

(1) If the question can be answered from one of the 3. On how many days did Raju's dad go to only one statements alone but not from the other. temple? (2) If either statement alone is sufficient to answer (1) 204 (2) 214 (3) 220 (4) 240 the question. 4. If A, B, C, D, E, F and G are distinct single digit (3) If both the statements together are sufficient but natural numbers from 1 to 7such that A + B + C = either statement alone is not sufficient. C + D+ E = E + F + G = 11, then how many (4) If the question cannot be answered even by ordered pairs (C, E) exist which satisfy the given combining both the statements relation? (1) 0 (2) 2 (3) 4 (4) 8 1. Pramod bought a new car after selling his old car. If the cost of the old car was 40% that of the new CASELET

car, find the price of the new car. Directions for questions 1 to 3: Answer the following Ι. He borrowed an amount which was equal to questions based on the statements given below. 60% of the cost of the old car from his friend and raised the remaining amount by (i) There are three buildings on each side of the withdrawing from his personal savings account. road. ΙΙ. His total personal savings were `3,00,000 (ii) These six buildings are labelled as A, B, C, D, E and F. (iii) The buildings are of different colours, namely, Violet, Indigo, Blue, Green, Yellow, and Orange. (iv) The buildings are of different heights. 22 (v) E, the tallest building, is exactly opposite to the (1) P, S and U (2) S and T Violet coloured building. (3) T and U (4) S, T and U (vi) The shortest building is exactly opposite to the (5) S and U Blue coloured building. (vii) F, the Green coloured building is located 7. The team(s) with the most wins in the event is (are): between A and D. (1) p (2) P and R (3) U (viii) C, the Yellow coloured building is exactly (4) T (5) Q and T opposite to A. Directions for questions 8 to 12: Answer the following (ix) B, the Blue coloured building is exactly opposite questions based on the information given below. to F. (x) A, the Orange coloured building, is taller than C, Anand, Bala and Chandu are three professional but shorter than B and D. traders who traded in gold in the commodities market. Anand followed the strategy of buying at the opening 1. What is the colour of the building diagonally of the day at 10 a.m. and selling the whole lot at the opposite to the Yellow coloured building? close of the day at 3 p.m. Bala followed the strategy (1) Orange (2) Indigo (3) Blue of buying at hourly intervals: 10 a.m., 11 a.m, (4) Violet (5) None of these 12 noon, 1 p.m. and 2 p.m. and selling the whole lot at the close of the day. Further, he buys an equal 2. Which is the second tallest building? quantity (by weight) in each purchase. Chandu (1) A (2) B (3) C followed a similar pattern as Bala but his strategy is (4) D (5) Cannot be determined some what different. Chandu total investment amount is divided equally among his purchases. The profit or 3. What is the colour of the tallest building? loss made by each investor is the difference between (1) Violet (2) Indigo (3) Blue the sale value at the close of the day less the (4) Yellow (5) None of these investment. The return for each investor is defined as the ratio of the profit or loss to the investment amount Directions for questions 4 to 7: Answer the following expressed as a percentage. questions based on the information given below. 8. On a day of fluctuating prices, the price of gold Six teams (P, Q, R, S, T and U) are taking part in a ends with a gain, i.e., it is higher at the close of cricket tournament. Matches are scheduled in two the day compared to the opening value. Which stages. Each team plays three matches in stage-Ι trader got the maximum return on that day? and two matches in stage-ΙΙ. No team plays against (1) Bala (2) Chandu the same team more than once in the event. No ties (3) Anand (4) Bala or Chandu are permitted is any of the matches. The observations (5) Cannot be determined after the completion of stage-Ι and stage-ΙΙ are as given below. 9. Which one of the following statements is always true? Stage Ι: (1) Anand will not be the one with minimum return. • One team won all the three matches. (2) Return for Chandu will be higher than that for Bala. • Two teams lost all the matches. (3) Return for Bala will be higher than that of • S lost to P but won against R and U. Chandu. • T lost to Q but won against R and U. (4) Return for Chandu cannot be higher than that • Q lost at least one match. of Anand. • U did not play against the top team of stage-Ι. (5) None of the above Stage ΙΙ: • The leader of stage-Ι lost the next two matches. 10. On a “boom” day the price of gold keeps rising • Of the two teams at the bottom after stage-Ι, one throughout the day and peaks at the close of the day. team won both matches, while the other lost both Which trader got the minimum return on that day? the matches. (1) Bala (2) Chandu • One more team lost both matches in stage-ΙΙΙ. (3) Anand (4) Anand or Chandu (5) Cannot be determined 4. The two teams that defeated the leader of stage-Ι are: One day, two more traders, David and Emma joined (1) U and S (2) T and U (3) Q and S Anand, Bala and Chandu for trading in gold. (4) T and S (5) U and Q David followed a strategy of buying equal quantity of gold at 10 a.m., 11 a.m. and 12 noon and selling the 5. The only team(s) that won both matches in same quantity at 1 p.m., 2 p.m. and 3 p.m. Emma on stage-ΙΙ is (are): the other hand followed the strategy of buying using all (1) Q (2) T and U her money at 10 a.m. and selling all of them at 12 noon (3) P, T and U (4) Q, T and U and again buying using all her money at 1 p.m. and (5) Q and U again selling them at the close of the day at 3 p.m. At the close of the day the following was observed. 6. The teams that won exactly two matches in the event are: (i) Anand lost money in the transaction. 23 (ii) Both David and Emma made profits. Directions for questions 16 and 17: Akira and Aroki (iii) There was an increase in the price of gold during read four books each from among A, B, C, D, E, F, G the closing hour compared to the price at 2 p.m. and H such that each of the eight books was read by (iv) The price of gold at 12 noon was lower than the exactly one person. Further the following information closing price. was known. The person who read D, also read F. Books A and B 11. The price of gold was its highest at were not read by the same person. The person who (1) 10 a.m. (2) 11 a.m. (3) 12 noon read book F, did not read book C. (4) 1 p.m. (5) Cannot be determined 16. If Akira read books E and G, then Aroki did not 12. Which of the following is necessarily false? read book (1) The price of gold was not at its lowest at 2 p.m. (A) D (B) F (C) H (D) C (2) The price of gold was at its lowest at 11 a.m. (3) The price of gold at 1 p.m. was higher than 17. If books C and E were not read by the same the price at 2 p.m. person, then which of the following two books (4) The price of gold at 1 p.m. was higher than was definitely read by the same person the price at 12 noon. (A) A and C (B) B and C (5) None of the above (C) G and H (D) Cannot be determined

Directions for questions 13 to 15: Select the Directions for questions 18 to 25: Select the correct alternative from the given choices. correct alternative from the given choices.

13. Four persons A, B, C, and D on a tour to a hill 18. Nine workers – W , W , W , W , W , W , W , W , station booked four consecutive rooms among 1 2 3 4 5 6 7 8 and W are to be allotted work in four shifts – 101, 102, 103 and 104 (all the rooms being on 9 Morning, Afternoon, Evening and Night with not the same side in that order) in a hotel for their more than three workers in the same shift. stay there. A had a dispute with B and did not The workers are allotted the shifts as per the want to stay in a room adjacent to him. C being a following conditions. childhood friend of D booked a room adjacent to Each worker can be allotted work in only a single D who in turn booked the room adjacent to B. If B shift. booked an odd numbered room, then which room W and W do not work in the same shift. did C book? 1 2 W is to be allotted a shift, earlier than W but (A) 101 (B) 102 (C) 103 (D) 104 3 6 later than W2. 14. Four girls Dolly, Molly, Polly and Kelly appeared W2 is to be allotted the morning shift and W4 is to for their semester paper on Mass Communication be allotted a shift which is two shifts after W1. in which two of them failed. When asked as to W5 and W7 are both allotted the same shift and it who passed in the exam, they gave the following is one shift earlier than the shift allotted to W6. replies. W9 is allotted the same shift as W3. Dolly: I did not fail in the examination W8 cannot be allotted work in which of the Molly: I passed in the examination and so did Kelly. following shift? Polly: Only one among Dolly and Kelly failed in (A) Morning (B) Afternoon the examination (C) Evening (D) Night Kelly: Only one among Polly and Dolly passed in the examination. 19. Four players – A, B, C and D are the members of a If it is given that exactly three of them were telling cricket team. They are to lead their team in the the truth, then find the person who was lying. upcoming tournament where they will be playing (A) Dolly (B) Molly (C) Polly (D) Kelly five matches: Match1, Match 2, Match 3, Match 4 and Match 5 to be held one after the other with 15. In IIBM, a reputed B school with a total strength each match having two leaders, one as captain and of 270 students, every student opted for at least the other as vice captain. No person can Captain one specialization among the three – Finance, the side in two consecutive matches and neither HR and Marketing. The number of students who can the same person be the vice captain on opted for all three was 37.5% of those who opted consecutive matches. These players are to lead for exactly one. The number of students who did their team subject to the following conditions. not opt for Finance was 50% of these who opted (1) B was the captain of the side for Matches for Finance which in turn was 25% more than 1 and 3. those who did not opt for HR. The number of (2) C can be the vice captain only if A is the students who opted for only marketing and HR captain. was equal to those who opted for only Finance (3) D refused to lead the team as captain if A or 1 and Marketing and also 33 /3% of those who B led the team as captain in the preceding opted for all three categories. If the number of match. students who opted for only Finance and HR was (4) Each of the four players was the captain and 50% of those who opted for only Finance, then the vice captain in at least one of the five how many students opted for exactly two of the matches. three specializations? (A) 36 (B) 72 (C) 45 (D) 54 24 Which of the following statements is true? 24. A race is held on three days and there are three (A) C was the vice captain in Match 5. drivers – Schumi, Sebastian and Mclaren. (B) B was the Vice captain in Match 4. On each day they are ranked from one to three in (C) C was the vice captain in Match 2. the order in which they finish the race. On any (D) A was the captain in Match 5 day, Schumi is always ranked ahead of Mclaren. No driver secures the same rank on more than 20. Each of the three persons – A, B and C have to two days. Which of the following must be false? buy three household appliances – AC, (A) Sebastian wins the race on exactly two days. Refrigerator and Water Purifier on three days – (B) If Sebastian wins the race on Day 1, then he Monday, Tuesday and Wednesday. No two is ranked below Mclaren on the remaining persons buys the same type of appliance on any two days. single day. B buys the Refrigerator on Tuesday, (C) Sehbastian comes last on exactly two days. while he has to buy the Water Purifier before (D) Sebastian beats Mclaren on all the three days. buying the AC. Which of the following is true regarding A? 25. A company has two branches, one at Kukatpally (A) A buys the Water Purifier before the and the other at Narayanguda. Six persons – Refrigerator. Tom, Raj, Ryan, Mokambo, Sashi and Govind (B) A buys the AC before buying the Water Purifier. have to work in these branches. However, no (C) A buys the AC before buying the Refrigerator. person works in both the branches. Each branch (D) A buys the Refrigerator before buying the AC. has three employees. Further, it is known that (1) Tom & Raj do not work in the same branch. 21. The following table gives the average runs scored (2) Sashi and Govind works in the same branch. by four players in all the matches in a year. Which of the following must be true? Player Average Runs (A) If Ryan works in the Narayanguda Branch, Sachin 48 then Mokambo does not work in the Dravid 50 Kukatpally branch Mongia 40 (B) Both Tom and Mokambo work in the same Hussey 42 branch. Sachin & Mongia 44 (C) Both Govind & Raj work in the same branch. Dravid & Hussey 46 (D) More than one of the above.

Sachin played more number of matches than Directions for questions 26 to 28: The analysis of Dravid. If n is the average runs scored by all four the way the three star players of Mumbai Indians players, then what must be true about x? made runs is given below. The runs made by Pollard, (A) 44 < x < 45 (B) 45 < x < 46 Dumminy and Bhajji consist of three types of shots (C) x = 45 (D) None of the above “straight drive”, “pull shot” and “others”.

22. In an international maths olympaid, there were (i) The total runs made by Pollard is 40 more than seven questions. The marks scored by two that made by Bhajji. candidates in seven questions (though not (ii) Pollard scored 20% of his runs through the ‘pull necessarily in the same order) from Q1 to Q7 are shot’. as follows (iii) The runs made by Dumminy is the average of the runs made by Pollard & Bhajji. Ramesh: 26, 30, 34, 42, 46, 50, 54 (iv) Bhajji scored 25% of his runs, i.e., 20 runs, Sanjay: 13, 34, 42, 46, 50, 54, 62 through he ‘pull shot’. One of the questions was declared invalid (v) The runs scored by Dumminy through ‘others’ is (after the exam and the marking was over). 15% of the sum of the total runs scored by The average marks of both the candidates in the Pollard and Bhajji. remaining six questions were calculated. Both (vi) The runs scored by Dumminy through “straight candidates scored same marks in the “invalid drive” is 60% of the total runs made by him. question”, and so as a result of its exclusion, the (vii) Each player scored at least one run through each average score of one candidate increased while type of shot. that of the other candidate decreased. What was the marks scored by each candidate in the 26. Find the maximum possible difference between “invalid question”? the runs scored by Pollard through ‘Straight drive’ (A) 42 (B) 46 (C) 34 (D) 50 and that by Dumminy through ‘Pull shot’. (A) 110 (B) 86 23. (1) If A wins, then B wins. (C) 50 (D) None of these (2) If B wins, then C does not win. (3) Only if D wins, then at most one of A or C wins. 27. The runs scored by Bhajji through ‘straight drive’ is If there are only four players A, B, C and D, then x% of the total runs scored by all the three players put together. Find the maximum possible value of x. which of the following must be true? 2 (A) D wins (B) D does not win (A) 20 (B) 30 (C) 19 (D) 19 /3

(C) B wins (D) A does not win 25 28. Find the runs scored by Bhajji through ‘others’. Biology and were ranked from 1 to 5 (1 being the (A) 50 highest and 5 being the lowest) in each of the (B) 60 exams. The sum of the ranks obtained by them in (C) 40 the four exams were 11, 15, 12, 8 and 14 (D) Cannot be determined respectively in the same order where no two students got the same rank in any exam. Further Directions for questions 29 to 41: Select the it was known that correct alternative from the given choices. (1) Dimitry got the 4th rank in Physics and his ranks in Chemistry and Biology were the same. 29. (1) All shoes are pens. (2) The rank of Emmanuel in Chemistry was the (2) Not all pens are pencils. same as that of Dimitry in Mathematics. (3) All pens are chocolates. (3) The ranks secured by Ben in Mathematics (4) Not all chocolates are pens. and Chemistry was the same. Which of the following must be true? (4) Ben had the least rank in Physics and Cathy rd (A) Some chocolates are not shoes. got the 3 rank in Mathematics. (B) Some shoes are chocolates. (5) Emmanuel got distinct ranks in all four (C) Some pencils are not chocolates. subjects and did not get the best rank in any (D) More than one of the above. subject. Which of the following is definitely true. 30. Ramesh was gifted a wonderful watch by his (A) Adam’s rank in Maths was 2. father. When the watch was showing 4 p.m. (B) Adam’s rank in Physics was 2. Ramesh started for his friend Umesh’s house to (C) Adam’s rank in Chemistry was 4. show off his watch. He was driving at a constant (D) Cathy’s rank in Chemistry was 3. speed. His friends watch was showing 6:10 p.m. at the moment Ramesh arrived at Umesh’s 34. There are 120 families living in a housing society house. Ramesh immediately drove back at where each family owns at least one among a 5 Refrigerator, an Air Conditioner and LCD TV. th of the earlier speed. As soon as he reched 4 • 24 families have only a Refrigerator. home, he saw that the time shown by his watch • 20 families have only an Air Conditioner. was 7:45 p.m. By how many minutes is Ramesh’s • 26 families have only a LCD TV. watch faster/slower as compared to Umesh’s • At least 40 families own both Refrigerator as watch? well as Air conditioners. (A) No difference (B) 10 mins slower At most how many families own a refrigerator and (C) 10 mins faster (D) 5 mins slower an LCD TV but not an air conditioner? (A) 10 (B) 12 (C) 15 (D) 30 31. In a selection processes, each candidate has to appear for two types of tests – T1 and T2. 35. Five lecturers L1, L2, L3, L4 and L5 are to deliver 200 candidates failed in T2 while 300 failed in T1. lectures in a college on four consecutive days Ratio of the number of candidates who failed in from Monday to Thursday with two lectures to be both T1 and T2 to those who passed in both T1 delivered on each day. No lecturer is to deliver and T2 is same as the ratio of the number of more than two lectures in the given period. candidates who passed in T2 to those who Further it is known that, passed in T1. This ratio is an integral value. Find L1 delivers lectures on Monday and Thursday only. the number of candidates who passed in both the L3 delivers a lecture on a day only if L2 delivered tests. a lecture the preceeding day. (A) 100 (B) 50 L1 and L4 do not deliver lectures on the same (C) 20 (D) Cannot be determined day. Each lecturer delivered at least one lecture in the given period. 32. Four persons – W, X, Y, Z secured distinct ranks L3 and L4 do not deliver lectures on consecutive from 1 to 6 in four events – Swimming, Running, days. Cycling and Walking.

On which day did L3 deliver the lecture? Swimming Running Cycling Walking Total (A) Monday (B) Tuesday W 5 6 15 (C) Wednesday (D) Thursday. X 1 6 Y 5 2 4 14 36. A committee consisting of five members is to be Z 1 18 formed from five boys among B1, B2, B3, B4 and Which of the following was ranked 3? B5 and three girls among G1, G2 and G3. (A) X in Cycling There must be three boys and two girls in the (B) W in Swimming committee. Further it is known that, (C) Z in Running (1) If B1 is selected, then B2 cannot be selected. (D) More than one of the above. (2) If B3 is selected, then B4 cannot be selected. (3) Both G1 and G2 cannot be selected at the 33. Each of the five students Adam, Ben, Cathy, same time. Dimitry and Emmanuel wrote four exams, one (4) If B4 is selected then B5 must also be selected. each in Mathematics, Physics, Chemistry and (5) If B2 is selected, then G2 must also be selected. 26 The total number of ways in which the committee 41. Two teams, each with three members, are to be can be formed is selected from among the seven students – P, Q, (A) 5 (B) 4 (C) 3 (D) 6 R, S, T, U and V for the Inter School Meet

consisting of a Debate and an Elocution contest. 37. Three products P1, P2 and P3 needs to be In addition it is also known that machined in two machines M1 and M2. A product cannot be machined in two machines at the same (1) P was the only student who represented his time and the entire machining of a product in a school in both the contests. machine must be completed in one go. A product (2) If Q was selected for Debate, then R must be can be machined in the machines in any order selected for the Elocution contest. (i.e., M1 before M2 or M2 before M1). The duration (3) Both S and T cannot be selected for the (in hours) for which the products needs to be same event. machined in each machine is given in the table (4) V represented his School in Debate where as below. U was selected for one of the two categories. M1 M2 (5) If R was selected for the Elocution contest, P1 3 2 then U cannot be selected for Debate. P2 4 3 Which of the following statement is definitely P3 2 5 false?

If the total time taken to finish machining all the (A) Both U and Q cannot be selected for the three products is the minimum, then which of the same event. following is definitely false. (B) If both V and U are selected for the same (A) Product P3 is machined in M1 before Product P2 event, then Q must be selected. (B) Product P2 is machined in M2 before Product P1. (C) If S is selected for Debate then either R or T (C) Product P2 is machined in M1 before Product P1. or Q must be selected for Elocution. (D) None of these. (D) None of these.

38. In a group of 30 members belonging to a Sports NETWORKS Club, each member played at least one of the three games from football, cricket and hockey. Directions for question 1 and 2: The following 18 members played at least two games. If the network gives the bus routes of APSRTC in number of members playing exactly one game was Hyderabad for its new A/C buses introduced in the three times that of those playing all the three games, previous month. Any passenger boarding a bus is then how many played exactly two games? charged `5 as local service charge and `8 as fixed (A) 12 (B) 14 (C) 16 (D) 10 ` charge in addition to a charge of 4 per km. Further it 39. Ten athletes M1 to M10 competing in an athletics is also known that meet represented five countries among UK, Germany, France, Switzerland and Turkey with (1) a bus does not visit the same city more than each country being represented by two athletes. once. Further it was known that M1 and M3 represented (2) between any two cities only one mode of the same country. transport is available, i.e. the bus. Both M2 and M4 were either from France or from (3) in the network shown, the values in brackets Germany. denote the distance in kilometers. M and M were from different countries. 5 9 M belonged to UK and so did M 6 8 B (5) C M was from Turkey and M did not belong to 7 9 France or Switzerland. (6) (4) Which country did M belong to, if M did not 10 5 (5) (7) (3) belong to France or Switzerland? H

(A) France (B) Turkey G A D (1) (3) (C) Germany (D) Switzerland (1) (8) (2)

E (3) F 40. Each of the four friends Sneha, Shikha, Sushma

and Sushmita bought a birthday present for their 1. Find the minimum cost incurred by a person to common friend Rahul. The gifts bought by them were a shirt costing `1200, a tie costing `800, travel from A to H. ` ` ` ` trousers costing `2000 and a pair of shoes (1) 53 (2) 75 (3) 61 (4) 73 costing `2800 (not in the same order). The sum of the costs of the gifts bought by Sneha and 2. If the road connecting A to E is under repair, then Sushma was equal to that of the gift bought by what is the minimum cost incurred by a person to Shikha. Further the difference between the cost travel from A to H? of Sushma’s gift and Sushmita’s gift was equal to (1) `57 (2) `59 (3) `61 (4) `73 the cost of Sneha’s gift. What was the gift bought by Shikha?

(A) Shirt (B) Trousers (C) Tie (D) Shoes 27 QBR (Miscellaneous) Directions for questions 2 and 3: There are two different investment schemes Ι and ΙΙ yielding Directions for question 1: Select the correct different percentage returns based on the market alternative from the given choices. conditions. Market conditions are catagorised into

1. A group of 450 persons was tested for HIV three types, namely bearish, steady and bullish. The infection, but there was an error in the testing probabilities of the market conditions and the process due to which four types of results were respective yield percentages for the two schemes are observed. The results identified the following four given below categories of persons. C1: Persons infected with HIV but reported Scheme Ι negative in the test. C2: Persons not infected with HIV but reported Market conditions Probabilities Yield percentage positive in the test Bearish 0.25 –30 C3: Persons not infected with HIV and reported Steady 0.55 80 negative in the test Bullish 0.20 100 C : Persons infected with HIV and reported 4 ΙΙ positive in the test Scheme If it is known that the results of 275 people were correctly reported and that the number of infected Market conditions Probabilities Yield percentage people is 50% that of non-infected people, find the Bearish –10 difference in the number of people under Steady 0.4 60 categories C2 and C4. Bullish 100 (1) 11 (2) 25 (3) 91 (4) 100 2. If the total yield from both the schemes is the DI (Miscellaneous) same, what is the probability of the market ΙΙ Directions for question 1: A company manufacturing condition being bearish for scheme ? cricket balls incurs a manufacturing cost of `50 per (1) 15% (2) 20% (3) 25% (4) 35% ball. The sale of the number of balls decreases linearly such that the number of balls sold decreases 3. If the probabilities for the market conditions being by 20 for every `2 rise in the selling price. It is known bearish, steady and bullish for scheme Ι are 0.2, ` that when the selling price of each ball is 59, the 0.45 and 0.35 respectively instead of what is number of balls sold is 700. given in the table, then for the same yield 1. Which of the following will give maximum profit? percentage (as given in the table), what would be Selling price Number of balls sold the percentage increase in the total yield from ` (1) 89 400 scheme Ι? ` (2) 93 360 (1) 8.9% (2) 9.1% (3) 10.2% (4) 15% ` (3) 85 440 (4) `99 300

Directions for questions 4 and 5: The following figure gives the per capital income and the Happiness Quotient of fifteen countries in the world vs the Per capita income. (Only for families with 3 or more members)

1.0 • J G • O • L • F • 0.8 H • B • N • D • 0.6 A • M 0.4 • E • • C • I • K 0.2

Happiness Quotient 3000 4000 5000 6000 7000 8000 9000 10000 11000 12000 13000 14000

Per capita Income (in US $)

The following table gives the names of the countries represented in the above figure.

A-Taiwan E-China I-Philippines M-Thailand F-Czech B-Australia J-Poland N-Singapore Republic C-Japan G-Israel K-Slovakia O-Austria D-South H-Brazil L-Latvia Korea

28 4. Which of the following gives the list of the 5. Which of the following lists indicate “high countries which indicate “Low per capita Income happiness quotient with high per capita income”. with low happiness Quotient” (1) Australia Latvia, Austria (1) China, Thailand, Slovakia (2) Australia, South Korea, Austria (2) China, Slovakia, Brazil (3) Australia, Austria, Israel (3) China, Poland, Brazil (4) Australia, Latvia, Czech Republic. (4) China, Slovakia, Poland

Direction for question 6: The selection of a candidate into a B - school consisted of evaluating various factors - Written score, Group Discussion, Essay writing, Interview and Work experience. The B - school assigns some weightage to each of these factors. The cumulative score of a student is the sum of the product of the scores of the student attained in these factors and the weightage assigned to the respective factor. The following diagram gives the scores of two students Rahul and Ramya and also the weightage assigned to each factor. WRITTEN SCORE 30%

5

4 3 2 WORK EXPERIENCE 10% GROUP 1 5 5 4 3 4 DISCUSSION 3 2 1 1 2 25% 1 1 2 2 Rahul 3 _ _ _ _ _ Ramya 3 4 4 5 5

ESSAY WRITING 10% INTERVIEW 25% 6. What is the difference in the cumilative values of the scores obtained by Rahul and Ramya? (1) 0.1 (2) 0.2 (3) 0.25 (4) 0.15

Directions for questions 7 and 8: In a game of cards Expected pay-out for any gambler is the weighted played by two gamblers Raju and Ramu, the payout average of probability of occurrence and payout. when a card out of King, Ace and Joker turns up, is given below along with the probability of occurrence. 7. What is the expected pay-out for Raju? (1) 40 (2) 48 (3) 60 (4) 70 Pay out Probability of Card Gambler ` (in ) occurrence 8. If the probability of getting a King and a Ace for King Raju 80 0.5 Ramu is interchanged then what will be the Ace Raju 40 0.3 percentage decrease in pay-out after the Joker Raju −20 0.2 interchange as compared to the original pay- King Ramu 80 0.5 out? Ace Ramu 60 0.3 (1) 7.4% (2) 7.2% (3) 6.25% (4) 12.5% Joker Ramu −20 0.2

Direction for question 9: An agency "Book a Hotel" offers its customers an incertive to book a hotel room in advance. It also offers its customers a half-a-day tour and also a tour of the Hunters Valley. The data is given in the following flow chart: Hotel Booking

3 star 5 star

S/o $600 D/o $1100 S/o $1000 D/o $1900

CWB $250 CWNB $125 CWB $350 CWNB $200

City tour Yes($ 40)

Note: S/O – Single Occupancy No D/O – Double Occupancy CWB – Child With Bed Hunters Valley Yes($ 35) CWNB – Child With No Bed.

No 29 9. If Ramu, an employee of EMIT, books a hotel 12. What is the total sum of the numbers in column room in a 3 star hotel, goes on a city tour and 'a'? visits the Hunters Valley, then what is the (1) 208 (2) 312 (3) 216 (4) 373 amount spent by Ramu? (1) $685 (2) $665 Directions for questions 13 and 14: Ram, a saree (3) $675 (4) $655 seller bought 400 sarees of 3 different types (Kanchipattu, Benarasi and Mangalagiri) in the ratio 10. 3 groups for 3 different modules have to be 5:3:2. He sells the sarees based on market demand. made from 4 junior software engineers (JSE) P, On the 1st day he sells 20% of the total volume. On Q, R, S and 4 senior software engineers (SSE) the 2nd day he sells 50% of the total volume. On the X, Y, Z, W. 3rd day, he sells 30% of the total volume. The cost of (1) P and S are in the group. each Kanchipattu, Benarasi and Mangalagiri saree is (2) X and Y cannot be in the same group as Q. `350, `400 and `375 respectively. On each day he (3) R is a member of a group which has 3 sells the sarees of each variety in the same ratio as people. he bought. (4) R, Z, W are males. (5) There has to be a male in every module. 13. If on the 3rd day he sells Benarasi saree for a (6) There has to be a JSE and a SSE in every price which is more than 20% than their cost module. price whereas the other two sarees are sold at The person who is in the group having 3 their cost price, then find the total amount members is received by Ram by selling the sarees on the 3rd (1) X (2) Q (3) Z (4) W day? (1) `45280 (2) `47280 Directions for questions 11 and 12: The following (3) `51280 (4) `49280 table shows some integers in a 5 × 5 grid. 14. It is known that 25% of the sarees were slightly a b c d e damaged. If Ram managed to sell the sarees a 4 13 which were slightly damaged at 20% loss, then b 8 12 what is the amount made by Ram by selling the c 14 10 damaged sarees. [consider the ratio of the d 17 12 7 damaged sarees in the three categories is the e 8 same as the ratio in which he bought] ` ` (1) 37500 (2) 35000 ` ` The numbers in the blocks are placed according to (3) 29600 (4) 28000 the following conditions. 1. The number in column 'b' and row 'd' is 1/3 the 15. The population of China in 2009 is 1.6 billion and sum of the numbers in column 'b'. it is expected to grow at 12% every year till 2020. 2. The numbers in column 'a' are squares of prime If in 2020 the population of China is 15% of the numbers in ascending order starting with the total population of the world, then find the total first odd prime number. population of the world in 2020 approximately? (1) 30 billion (2) 37 billion 11. Which of the following is the value of the (3) 45 billion (4) 51 billion number in column 'b' and row 'b'? (1) 23 (2) 19 (3) 17 (4) 16

Direction for question 16 and 17: The following graph gives the visibility index of a certain number of people:

1

• I • • R S 0.8 • K • D • C • • • A • Q • J X• L T 0.6 • • B • • M W • U • H F 0.4 • N • • Y V • • P • G • Z • E O 0.2

Index Visibility 0

16. For what percentage of the total number of 17. For which pair of persons the visibility index are persons is the visibility index more than U? equal? (1) 50% (2) 54% (3) 60% (4) 64% (1) F, M (2) C, L (3) R, S (4) A, T

30 18. Six friends A-F play a game of cards. The winner of the game is found by adding the points allotted to the cards. The person having the highest number points is declared the winner of the game. The six friends are sitting around a circular table as given below

A B K-1 K-2 J-2 J-1 Card Points A-2 A-1 Q-1 Q-3 K-2 K - King 30 K-1 J-2 J-1 A-1 C J - Jack 20 F A-1 Q-1 K-1 Q-1 K-1 J-0 J-2 A - Ace 10 A-2 A-1 Q-2 Q-0 Q - Queen 5 E D

Who among the following is the winner? (1) A (2) B (3) C (4) E

Quant SI – CI (1) 54 (2) 60 (3) 62 (4) 69

Directions for question 1 and 2: A company offers Directions for questions 2 and 3: Out of 800 four schemes for investors to invest. The schemes persons living in a locality, 50% own a car, 60% can are as follows. speak in Hindi, and 500 can speak in English. 15% of the total number of persons living in the locality own a Scheme 1: Gives a return of 8% p. a. interest being car and can speak both in Hindi and English. Of the compounded annually people who own a car, 240 cannot speak in English. Scheme 2: Gives a return of 15% p. a. simple Out of the people who can speak in Hindi, 180 own a interest car. There is no one in the locality, who does not own Scheme 3: Gives a return of 10% p. a. interest being a car, cannot speak in Hindi and cannot speak in compounded semi-annually English. Scheme 4: Gives a return of 20% p. a. compounded annually but 10% of the amount at the 2. What percent of the people who can speak in end of each year is paid as English, can also speak in Hindi? administrative charges and only the (1) 48% (2) 54% (3) 72% (4) 75% remaining 90% is reinvested. 3. What proportion of the people in that locality do 1. If a person decides to invest an amount of not own a car or cannot speak in English? `20000 equally in scheme 2 and scheme 3, then (1) 0.44 (2) 0.64 (3) 0.72 (4) 0.8 after how many years will his interest from scheme 3 be more than that from scheme 2? 4. In a survey conducted among 120 families living (1) 7 (2) 9 (3) 10 (4) 11 in a locality regarding the daily newspaper they read among The Telegraph, The times of India 2. If a person invests his entire money equally in all and The Hindu, the following data was recorded. four schemes, then which scheme will fetch him The number of families reading The Hindu, The the maximum interest after 8 years? Times of India and The Telegraph were 64, 48 and (1) scheme 1 (2) scheme 2 45 respectively.21 families did not read any of the (3) scheme 3 (4) scheme 4 three newspapers whereas 9 families read all three newspapers. LA (Venn Diagram) How many families read exactly one newspaper? (1) 40 (2) 45 1. In a class all the students applied in at least one (3) 50 (4) Cannot be determined of the three examinations among CAT, FMS and XAT. 24 students applied for FMS, 32 students Directions for questions 5 and 6: 300 employees applied XAT and 36 students applied for CAT. work in a company. 20% of the employees are HOD's. 12 students applied for both FMS and XAT, 70% of the employees have weekly off on Sunday. 180 15 students applied for both XAT and CAT employees own a car. Half of the HOD's own a car. All whereas 9 students applied for both FMS and the HOD's have their weekly off on Sunday. 30 CAT. If the number of students who applied for all employees have a car but do not have their weekly off the three was 25% of those who applied for at on a Sunday. least two of the three examinations, then how many students were there in the class? 31 5. What percentage of the company HOD's own a Price Delivery time Pizza store car and have their weekly off on Sunday? (in `) (in minutes) (1) 5% (2) 10% (3) 15% (4) 20% A 120 30 B 155 25 6. How many employees are not HOD's and do not C 85 20 own a car? D 95 45 (1) 60 (2) 70 (3) 80 (4) 90 E 125 30 F 120 40 7. 300 students passed engineering and chose only one of the two PG courses – MBA and MS. Which of the following pizza stores got the third 70 students chose MS. In MBA, students chose lowest net score? at least one of the three specialisations among (1) A (2) D (3) E (4) F Finance, Marketing and HR. 20 chose general specialisation which includes all three Directions for questions 2 to 4: In the Recruitment specialisations. 100 students chose Marketing. process, a company asked the people who applied to 150 chose Finance. It is also known that the number of students who chose only Marketing write four objective papers each consisting of four and Finance is twice the number of students who questions. In the first paper, for every correct answer chose only HR and Marketing which in turn is 15 marks were awarded whereas for every wrong equal to the number of students who choose all answer 5 marks were deducted. In the second paper, three specialisations. How many students chose for every correct answer 20 marks were awarded and only HR specialization? for every wrong answer 15 marks were deducted, but (1) 60 (2) 50 (3) 20 (4) 30 if a person got all 4 questions correct, an additional 30 marks were awarded. In the third paper, for every 8. A few children visit a toy shop and buy some correct answer 25 marks were awarded and for every toys. 16 children buy toys of T & J, 26 children wrong answer 15 marks were deducted. In the fourth buy toys of C & H and 34 children buy toys of B & paper, 20 marks were awarded for every correct B. It is known that each child buys exactly two answer and 5 marks deducted for every wrong toys. How many children visited the toy shop? answer but if a student got three or more questions (1) 28 (2) 32 (3) 34 (4) 38 wrong, 50 marks were deducted from his total.

Directions for questions 9 and 10: A survey was 2. What is the least score obtained by a person in conducted to find out which of the four movies P, Q, these four papers, if he gets 4 wrong answers R and S is liked by most people. The number of across all four papers? people who like P, Q, R and S are 24, 36, 29 and 25 (1) 200 (2) 150 (3) 180 (4) 165 respectively. It is further known that – every person surveyed likes at least one movie 3. What is the difference between the highest marks – the number of people who like P and Q are 12. obtained by two persons one of whom wrote paper – the number of people who like R and P, Q and 1 and paper 3 and the other wrote paper 2 and R are 18 and 16 respectively. paper 4, each getting 2 wrong answers overall? – the number of people who like only P, R and S (1) 20 (2) 30 (3) 40 (4) 60 are 8 and those who like P, Q and R are 6 – people who like S also like R. 4. What is the least total score obtained by a person who got 2 questions wrong and attempted 3 9. How many people like only Q? papers? (1) 10 (2) 11 (3) 12 (4) 14 (1) 145 (2) 150 (3) 160 (4) 165

10. How many people like all 4 movies? Directions for questions 5 and 6: In a 9 seater van (1) 7 (2) 6 (3) 5 (4) 4 having 3 rows of 3 seats each, the seats in the first nd LA (Miscellaneous) row are numbered 1, 2 and 3, those in the 2 row are numbered 4, 5 and 6 and those in these last row are 1. Six pizza stores are ranked as per the price numbered 7, 8 and 9. The seats are arranged in a offered by them on a particular variety of pizza grid form with 3 rows and 3 columns. These seats are and the time taken by them to deliver the order. occupied by 9 persons from P, Q, R, S, T, U, V, W The store offering the least price is ranked 1 and and X not necessarily in this order. The seats are when ranked as per the delivery time, the store occupied subject to the following conditions.. which delivered the earliest was ranked 1. If more 1. Q the driver occupies seat 3 and both R and V than one store offered the same price or are in his column. delivered in the same time, then they are given 2. P is the son of W and occupies the seat in the the median of all the rankings. The price ranking same row as V and S. is given a weightage of 0.7 whereas the delivery 3. T occupies seat 5 and is in the same row as W time ranking is given a weightage of 0.3. Study and is not seated immediately in front of S. the following table carefully and answer the given 4. X and T are not seated in the same column. question:

32 5. If persons sitting in the seats swap their positions revisited in the route, i.e. oil can pass through a in the following order (1,4) (2,6) (4,6) (5, 7), (4, 5), particular tank only once? (7,2), then which of the following persons will be (1) 5 (2) 6 (3) 7 (4) 8 seated beside U? (1) X (2) R (3) S (4) T 11. Arun, Varun and Kiranmala, subscribed to three business magazines, among India-Day-to Day, 6. If swapping of seats is done in the order (1,2) India-Everyday and India-These-Days subject to (2,8) (9,2) (4,9), (7,5) (5,8) (8,4), then which of the following conditions: the following is true? 1. If Arun subscribes to India-Day-to-Day, then (1) S is in the fifth seat. Varun subscribes to India-Every-day. (2) W is in the ninth seat. 2. If Arun subscribes to India-Every-day, then (3) T is in the second seat. Kiranmala subscribes to India-These-Days. (4) P is in the second seat. 3. If Varun subscribes to India-These-Days, then Arun subscribes to India-Every-Day. Directions for questions 7 to 9: A survey was 4. If Kiranmala subscribes to India-Day-to-Day, conducted in four colonies A, B, C and D whose then Varun subscribes to India-These-Days. populations are in the ratio 3 :5 : 3 : 4, to find (i) the 5. If Kiranmala subscribes to India-Every-Day, proportion of residents who prefer watching movie to then Arun subscribes to India-These-Days eating out & (ii) the proportion of people who prefer 6. If Varun subscribes to India-Day-to-Day, then surfing the internet to chatting with friends. The Kiranmala subscribes to India-Every-Day. following was the data collected. From the above constraints, which of the following can be definitely concluded, if it is given that each Proportion of person subscribed for exactly one business Proportion of residents magazine. residents who who preferred (1) Arun subscribed to India-Day-to-Day preferred watching (2) Kiranmala subscribed to India-Every-day surfing movies (3) Varun did not subscribe to India-These-Days. Colony A 0.6 0.64 (4) More than one of the above. Colony B 0.45 0.55 Colony C 0.65 0.68 12. If it is given that Kiranmala did not subscribe to Colony D 0.48 0.75 India-These-Days, then which of the following magazines did Varun subscribe to? 7. If 61% of the residents in colony C preferred both (1) India-Day-to-Day watching movies and surfing the net, then what (2) India-These-Days percentage of the residents in colony C did not (3) India-Every-day prefer any of the two? (4) Cannot be determined (1) 30% (2) 28% (3) 25% (4) 35% 13. There are 6 baskets 1, 2, 3, 4, 5 and 6. Two 8. In which colony do the maximum number of baskets have gold, two have silver and the other residents prefer chatting with friends? two have nothing in them. They are placed one (1) A (2) B (3) C (4) D beside the other in a row. Rajini, a contestant, has to select one of the baskets and can get 9. In how many colonies is the number of residents whatever is present in the basket. No two baskets who prefer surfing the net more than the average having the same thing inside are kept side by number of residents in the four colonies who side. What is the probability that Rajini goes prefer the same? home by taking something? 1 2 1 1 (1) 0 (2) 1 (3) 2 (4) 3 (1) (2) (3) (4) 6 3 3 2 10. There are eight oil tanks A, B, C, D, E, F, G & H in a refinery by which oil is being transferred. Due 14. In a forest there are P tigers and a goat. Grass is to the lack of sufficient connecting routes, oil eaten by both the goat and the tigers but the cannot be transferred from any tank to any other tigers would rather prefer eating a goat. If a tiger tank but can only be transferred subject to the eats the goat then the tiger transforms into a goat following conditions. and hence runs the risk of being eaten by another (a) Oil can be transferred from A to E or F tiger. The tigers are intelligent and would not risk (b) From E oil can be transferred to A, C or D being eaten. (c) From D oil can be transferred to A, B or C (d) From F oil can be transferred to D or G Which of the following is true? (e) From H oil can be transferred to A or E (1) If P is odd, no tiger eats the goat. (f) from G oil can be transferred to E, C or H (2) If P is odd tiger close to the goat eats the (g) from C oil can be transferred to A or G goat. (h) from B oil can be transferred only to E. (3) For any value of P, the goat is not eaten. (4) For any value of P, the goat gets eaten. In how many ways can oil be transferred from tanks B to tank H if the same tank cannot be

33 Directions for questions 15 and 16: During his 16. It is known that on 6 days he went to learn only holidays Ram wanted to learn Tabla and Piano. Ram Tabla and not Piano, then on how many days in went to learn Tabla for 12 days and to learn Piano for all did Ram go to learn playing musical 8 days. instruments? (1) 8 (2) 10 (3) 14 (4) 16 15. If on 4 days he went to learn both the instruments, then on how many days in all did Ram go to learn playing musical instruments? (1) 8 (2) 10 (3) 14 (4) 16

Directions for question 17: The following gives the process of arranging words by a word arrangement machine. Read the pattern below and answer the questions.

Input Sunday Monday Tuesday Wednesday Thursday Friday Saturday Step 1 Saturday Sunday Monday Tuesday Wednesday Thursday Friday Step 2 Friday Saturday Sunday Monday Tuesday Wednesday Thursday Step 3 Thursday Friday Saturday Sunday Monday Tuesday Wednesday

17. Which step will be the same as the input? (1) Step 24 (2) Step 26 (3) Step 29 (4) Step 28

LA (Circular Arrangements) – The Delhi co-ordinator is not opposite the Chennai coordinator but is to the left of the 1. Eight persons A, B, C,D,E,F,G and H are seated Hyderabad coordinator around a circular table, such that each person has another person seated exactly opposite him. Who is opposite to the Delhi coordinator? B is seated opposite to E who is seated to the (1) Mumbai co-ordinator immediate right of G.D is seated between E and (2) Chennai co-ordinator C. If F is not seated adjacent to A, then who is (3) Bengaluru co-ordinator seated opposite to H? (4) Hyderabad co-ordinator (1) D (2) C (3) G (4) either C or D 4. Six employees P, Q, R, S, T and U of EMIT Pvt Ltd. go to a party. They sit around a circular table. 2. Ten persons P, Q, R, S, T, U, V, W, X, and Y are P and S do not sit together and R and T always seated around a circular table with equal distance sit together. In how many ways can they be between any two adjacent persons such that any seated? person can only see the person seated opposite (1) 12 (2) 24 (3) 6 (4) 18 him and the two persons seated on either side of that person (i.e on either side of the person LA (Distribution) seated opposite). Further the following information is known. P does Directions for questions 1 to 3: Twelve persons not want to see Q, R, S, or T but wants to see X and who are members of a rock band live in a building Y. having twelve floors from the 1st floor to the 12th floor U does not want to see R, S, T or V, but wants to with the ground floor and the basement being used see P, W and Q. for parking. All the persons live in distinct floors. T wants to see X and Y. Among them there are 4 singers, 3 guitarists, 2 Which of the following gives the correct seating drummers, 2 instrumentalists and 1 keyboard player. arrangement? There are seven male members in the group A, B, C, (1) P T R U V X Y S Q W D, E, F and G and 5 female members P, Q, R, S and (2) P W Q V Y X U R S T T. These members live in the building subject to the (3) P S T R U Y X VQ W following conditions: (4) P Q W V Y X R U S T 1. No two female members lived on adjacent floors 3. A company "XYZ Ltd" asks some of its Branch and no singer lived on floors at the extreme ends, coordinators to attend a Round Table i.e. the topmost floor or the bottom most floor. Conference. 2. D and R were both guitarists and lived on adjacent floors but neither of them lived on the The coordinators sit around a circular table. topmost or the bottommost floor. 3. P and C were both singers and had exactly one – The Mumbai coordinator sits to the left of the guitarist living on an adjacent floor. Bengaluru coordinator, who is opposite the 4. A was neither a drummer nor a guitarist whereas Hyderabad co-ordinator F was not a drummer. 34 5. T lived on a floor adjacent to at least one 6. In a group, the number of boys must not be drummer where as Q, the keyboard player lived more than the number of girls. on the 7th floor. Jagan is in the same group as 6. All the singers were 3 floors apart from each (1) Kaya and Kavita. other where as all the guitarists were at least 2 (2) Kaya and Kadambari. floors apart from each other. (3) Kavita and Kadambari. 7. There were 3 floors between Q and B, who was (4) None of these an instrumentalist, and there was only one female guitarist. Directions for questions 7 and 8: There are 8. Both the drummers were males and neither lived 5 cricketers, L, M, N, O and P from different countries in the topmost floor or the bottommost floor. (India, Australia, S.A, SL, England) and playing for 5 different teams in IPL (KKR, RCB, DC, DD, MI). 1. How many male singers were there? These cricketers are made to stand in a row for post (1) 1 (2) 2 (3) 3 (4) 4 match presentation. – L, who is from SL, does not play for MI and is 2. How many floors were above the floor on which standing at the extreme right end of the row. G lived? – The cricketer who is to the immediate left of N (1) 4 (2) 6 plays for KKR and neither of them are from (3) 8 (4) Cannot be determined Australia. – The cricketer who is from India plays for RCB, 3. Who lived on the second floor? and is adjacent to only one player P who plays (1) A (2) S for DC. (3) F (4) Cannot be determined – O is to the immediate left of L, and is from SA.

Directions for questions 4 and 5: Six couples got 7. What is the correct combination of country and married on different dates in the years between 1993 the team for which M plays? and 1999. Only two couples got married in the same (1) India, RCB (2) England, KKR year and only two couples got married in the same (3) England, MI (4) SA, KKR month. The couple who got married on April 23rd, got 8. The player from Australia plays for which team in married before the couple who got married on th the IPL? October 15 , but after the couple who got married on th (1) DD (2) MI (3) KKR (4) DC November 5 . There is only one couple who got married in 1993 and they got married in the same 9. Four faculty members A, B, C, and D visited a month as the couple who got married in 1998, there college on 4 consecutive days starting from being only one couple who married in 1998. The th Monday and taught 4 different subjects – Maths, couple who got married on October 15 was not the Physics, Chemistry and Biology. Further it is last couple to got married and the couple who got rd rd known that A and C went to the college on married on January 3 was not the 3 couple to have consecutive days and taught Biology and got married. Two couples got married on the same Chemistry respectively. B went to the college on day in consecutive months but not in the same year. the last day and did not teach Physics. If Physics was taught immediately after Chemistry then 4. Both the couples who got married in the same when was Biology taught? month were married in the month of (1) Monday (2) Tuesday (1) October. (2) April. (4) Wednesday (3) Thursday (3) November. (4) January. Directions for questions 10 and 11: In a hockey 5. What was the least difference between the tournament, 15 teams from ‘A’ to ‘O’ participated. The marriage dates of any two couples (in days)? teams are arranged in ascending order of the points (1) 49 (2) 78 they scored in the tournament. (3) 160 (4) None of these The following information is know. 6. From a group of nine friends consisting of three boys Jalan, Jagan, and Jeevan and six girls (a) Team L scored the least number of points, i.e 96 Kekul, Kokila, Kavya, Kavita, Kaya and (b) Team I scored the maximum number of points, i.e Kadambari, three groups each consisting of three 364 members are formed from the above persons (c) Team D, team G and team C were placed 4th, 7th subject to the following conditions: and 10th respectively with 116, 182, and 218 1. Kavya and Jeevan should be there in the points. same group. (d) Team N and team B got 108, 165 points less than 2. Kavya and Kekul should not be there in the Team F. same group. (e) The points scored by Team 'O' = 251 3. Kokila wants Jalan in her group. (f) Team A got 361 points. 4. Kaya and Kavitha must be in the same (g) Team H got 4 points more than team M and team group. E got 18 points less than Team N. 5. Jalan prefers not to be in the same group as Kekul. 35 (h) The sum of the total points for the following But at the time of the seminar, some of the students positions are as follows. were missing. So the teacher had to change the order 1 + 2 + 3 = 302; 4 + 5 + 6 = 412; 7 + 8 + 9 = 590; of making presentations. 10 + 11 + 12 = 753; 13 + 14 + 15 = 1046 – X presented before T. (i) The ascending order of the teams is O, J, F, OJF. – R presented before V but after U. – Q presented after W, P and S 10. What is the difference in the points obtained by – W presented the seminar before U and X but teams J and K? after P and S. (1) 136 (2) 144 (3) 151 (4) 161 14. Which among the following is the group of 11. What is the position of team E? students who presented their seminar in the 1st (1) 7th (2) 8th (3) 9th (4) 10th group? (1) W, P, S (2) W, U, X Directions for questions 12 and 13: Seven friends (3) P, S, U (4) P, Q, R P, Q, R, S, T, U and V went to a restaurant. Each friend is wearing a different coloured shirt among 15. Who among the following cannot be the first Violet, White, Orange, Blue, Green, Yellow and Red. person to make his presentation in the 2nd group? They order seven different cool-drinks among Pepsi, (1) X (2) U (3) Q (4) V Thums up, Sprite, Mazaa, Coke, Mountain dew and Fanta. 16. Seven students P, Q, R, S, T, U and V go to tuition in different subjects-Mathematics, Physics – P, who wears an orange shirt drinks Sprite. and Chemistry. Out of 7 students, 3 were girls. It – The friend who wears a green shirt drinks Pepsi. is further known that – R wears a red shirt. – R goes for Physics tuition – U wears a blue shirt. – P and U go to the same tuition – T drinks Thums up and Q drinks Coke. – Every tuition is attended by at least 2 people – The friend who wears a green shirt, the friend – Q who goes to Maths tuition does not go to who drinks Fanta and V ordered the same dish. the same tuition as S. – The friend who wears the violet shirt drinks – T and V go to same tuition. Mazaa. Who among the following definitely goes to the tuition in which 3 students attend? 12. Who among the following drinks Pepsi? (1) S (2) T (3) V (4) Q (1) Q (2) S (3) T (4) V Quant ERPV 13. Which of the following is a correct combination of person, colour of shirt worn and the cool-drink he 1. Two auditoriums are designed to be constructed ordered? such that their floors are square shaped with the (1) V, violet, Maaza ratio of the sides of the squares being 1 : 2. A (2) U, blue, Fanta th group of N workers start working in A . After 1 (3) R, red, Mountain Dew 1 6 (4) Q, yellow, Coke 3 th of a day, of the workers working in A1 move 4 Directions for question 14 and 15: P, Q, R, S, T, U, over to A and start working. At the end of the V, W and X are students learning a foreign language. 2 day the work in A just gets completed and the and were divided into 3 groups of 3 each for the 1 work in A gets completed when N workers work purpose of presenting a seminar. 2 7 Group I P, Q, R for th of the next day as well. Find the value of Group II S, T, U 8 Group III V, W, X N. (1) 24 (2) 36 (3) 48 (4) Cannot be determined

36 Line + Bar graph

Directions for questions 1 to 4: The following bar graph gives the accident severity index and the type of vehicles responsible for accidents in the year 2005. A total of 75000 accidents occured during 2005.

50 45 45 40 40 35 34 35 30 28% 30 26%

25 18% 20 16% 15 12%

10 Accident Severity index index Severity Accident 5

(person killed per 100 accidents) per killed (person 0

Trucks Buses Cars 2 wheelers Others

1. How many persons were killed in 2 wheeler LA(Linear Arrangement) accidents? (1) 6780 (2) 7200 (3) 8400 (4) 9600 Directions for questions 1 to 3: Ten friends who stay in 3 different countries among USA, UK and 2. The ratio of people killed to people who got Australia meet at a reunion party in their school in injured in the accidents is the highest for India. They stand in a row according to their roll (1) 2 wheeler accidents numbers in the school. There are 5 male friends (2) Other types of accidents among P, Q, R, S and T and 5 female friends among (3) Car accidents F, G, H, I and J. 5 friends stay in USA, 3 stay in the (4) Truck accidents Australia and 2 stay in the UK.

3. How many persons were injured by car – No two friends from USA stand next to each accidents? other. (1) 6400 (2) 7800 (3) 4200 (4) 7200 – P is from Australia and stands in between I and J. – The friends from UK are at least 4 places away 4. By how much is the number of persons killed in from each other. truck accidents more than the persons injured in rd – S is from UK and is 3 in the row from the left accidents caused by other types of vehicles? end. (1) 1440 (2) 1720 (3) 1480 (4) 1680 – The extreme ends are occupied by the friends DI (Distribution) who stay in USA. They positions are of different genders. th th th Directions for questions 1 and 2: Atul has 6 policy – F, Q, G stand in the 7 , 6 and 5 from the right accounts which are going to mature in between the end of the row respectively. Two of these stay in years 1997 to 2002 (both the years included). Two the USA. policies have their maturity in the same month whereas – R is adjacent to I. two policies have their maturity in the same year. One of the policies matures on Feb 29th. It matures before 1. How many female friends stay in the USA? the policy which matures on January 10th but after the (1) One (2) Two policy which matures on August 8th. The first policy (3) Three (4) Four matures on August 24th 1997. The policy which matures on September 17th matures immediately 2. How many friends stand in between friends from before the policy which matures on May 21st which UK? inturn is the last policy to mature. (1) Three (2) Four (3) Five (4) Six 1. Which of the following policies is the 3rd to mature? 3. Who among the following is the male friend from the (1) Feb 29th, 2000 (2) Jan 10th, 2001 USA? (3) August 8th, 1999 (4) August 8th 1998 (1) Q (2) T (3) R (4) Both A and B 2. In which of the following years did Atul receive money from 2 policies? Directions for question 4 and 5: Seven friends (1) 2000 (2) 2001 Pradip, Qureshi, Raju, Shyam, Tarun, Uttam and (3) 1999 (4) 2002 Vamsi stand in a row. It is further known that 37 (i) Shyam is at the extreme left end of the row. 1. Which student got the highest marks? (ii) Qureshi is two places to the right of Tarun. (1) A (2) B (3) C (4) E (iii) There are exactly 2 persons in between Uttam and Vamsi. 2. Which two students got the same marks? (iv) Raju has the same number of people to his left (1) B and D (2) B and C as to his right. (3) A and D (4) C and D

4. Who is standing 3 places away to the left of 3. Which of the following is the descending order of Qureshi? the students according to their marks? (1) Tarun (2) Shyam (1) ECBDA (2) EACDB (3) Pradip (4) Raju (3) EBADC (4) ECADB

5. Who is standing to the immediate right of Directions for questions 4 and 5: Five movies P, Q, Shyam? R, S, T were set to release on every Friday of a (1) Qureshi (2) Uttam month. P was neither the first nor the last movie to be (3) Pradip (4) Vamsi released. Q was released immediately before R and there were at least 2 movies to be released before R. LA (Sequencing) There was one movie released in between S and T (Assume that no movie was scheduled to release Directions for questions 1 to 3: Five students A, B, during this period apart from these 5) C, D and E are arranged according to their marks under the following conditions 4. Which of the following movies was released last? (1) S (2) T (3) R (4) Q – C got more marks than A but less than E. – D got less marks than 3 other students. 5. How many movies were released before P? – Two students got the same marks. (1) 1 (2) 2 (3) 3 (4) 4 – The first and the last ranked students did not get the same marks as any other student.

Line Graph + Table

Directions for question 1: The following line graph gives the details of the production of different plants per unit area: 50 Steel Coffee 40 Coffee – –Tea Tea 30 Rubber –– Rubber

20 – – Steel

10

0

1996 1997 1998 1999 2000

Profit/ton = Revenue/ton – Cost/ton

The following table gives the cost and revenue obtained by these plants per ton in 2000.

Coffee Tea Rubber Steel Cost/ton `12340 `13460 `8900 `15800

Revenue/ton ``15610 `14280 `12300 `19960

1. Which of the following plants has the maximum profit in 2000? (1) Coffee (2) Tea (3) Rubber (4) Steel

38 SOLUTIONS FOR DATA INTERPRETATION REPLICA QUESTIONS THAT HAVE APPEARED IN CAT IN THE LAST 4 YEARS

TABLES 108800 Average gross pay = = 18133 6 Solutions for questions 1 to 3: 18133 −16000 ∴ percentage increases = ×100 1. Let the volume of data transfer in India and Singapore 16000 be 100 units each.  13% Choice (3) × Revenue from data transfer in India = 100 1 = $100 Revenue from data transfer in Singapore = 100 × 9 = $900 8. As after the mutual transfer, the average age of the 100 Finance department increase by one, it means that the Total revenue in India = 100 × = $1111 9 age of the person who came from the Marketing department was 20 years older than the age of the 100 Total revenue in Singapore = 900 × = $4285 person who was transferred from the Finance 21 department. Now after the transfer of the employee to ∴Total revenue in Singapore is about 4 times that in the HR department, as the average age of the India. Choice (5) employees left in the Marketing department remained the same, the age of the employee transferred to the 2. Revenue from data transfer as a percentage of total HR department, was 20 years younger than the revenue for India in 2010 = 27% average age, i.e., 36 – 20 = 16 years. Revenue from data transfer as a percentage of total ∴The new average age of the employees in the HR revenue for Sweden in 2010 = 36% department Let total revenue in India in 2010 be $200 and that in 46 × 5 + 16 ×1 246 Sweden be $100 = = = 41years Choice (3) ARDT of Sweden = $6 6 6

36 ∴Volume of data transfer in Sweden = = 6 9. The new average basic pay of employees in the HR 6 10,000 × 5 +12,000 × 2 +16,000 ×1 ∴Volume of data transfer in India = 6 department = 8 54 ∴ARDT in India = = 9 50,000 + 24,000 +16,000 90000 6 = = = 11250 8 8 9 −1 ∴ The percentage increase = ×100 = 800% The percentage change = 12.5% Choice (2) 1 Choice (3) Solutions for questions 10 to 13:

3. It can be seen that if the total revenue received is the 10. The drink must contain 10% minerals. As there are only same for the given pairs of countries, only UK and two drinks (A and C) with 10% minerals, the drink can Spain would have approximately the same volume of be prepared in only one way. As A and C have 30% data transfer. Choice (4) protein each, they can be mixed to form the drink. Choice (1) Solutions for questions 4 to 6: 11. None of the choices (1), (2) and (3) can be used to form 4. To get calls from all the colleges, Arun should have the drink with 10% fat and at least 30% protein. For C scored at least the highest value of cut-off in each and E to form the drink with 10% fat and at least 30% section, i.e., 44 + 44 + 45 + 44 = 177 and also at least protein, if they are mixed in the ratio x : y (say) the highest value of aggregate cut-off for any institute, x ()()50 + 0 i.e., 176. Choice (2) = 10, x : y = 1 : 4 x + y

5. The minimum aggregate marks to get calls from two 1()()200 + 4 100 600 ∴ cost per unit = = = 120 colleges is 171. If he scores 50 each in three sections 5 5 he needs to score at least 21 marks in the fourth Similarly the ratio for D and E is 1 : 3 and the cost per section. Choice (3) 800 unit is = 200 6. Four colleges have a cut-off for section C and the 4 remaining two colleges have a cut-off for section D. ∴ The cost per unit is the least for C and E. ∴ If a student misses the cut-off in these two sections, Choice (4) he/she would miss calls from all the colleges. The maximum possible marks such a student gets is 12. The drink should have at least 60% carbohydrate. 50 + 50 + 40 + 42 = 182. Choice (3) Further in the mixture formed by B, C and E, the proportion of B should be maximum and the other two Solutions for questions 7 to 9: should be minimum to get the lowest per unit cost. Among the given options only Choice (2) and (5) satisfy 7. The new gross pay of the employee transferred the condition having 60% carbohydrate and of these, 80 choice (5) has the lowest per unit cost. Choice (5) = 16,000 + × 16000 = 16,000 + 12,800 = 28,800 100 13. A and B when mixed in equal proportions, the protein The gross pay of the current employees in HR 30 + 20 department = 16000 × 5 = 80000 content will be only = 25%, which is less than New gross pay of the six employees = 80,000 + 28,800 2 = 1,08,800 required. D and E when mixed in equal proportion, the

1 5 + 45 Solutions for questions 18 to 21: carbohydrate content will be only = 25% which is 2 18. The costs of a refrigerator, an air conditioner and a less than required. Similarly B and E and C and D when music system in different countries are. mixed in equal proportion the combination will have less

than the required percentage of minerals and (’00 U.S. dollars) carbohydrate respectively. Only A and E when mixed in

equal proportion would yield a mixture with all the contents in the required amount. Choice (5) India ThailandMalaysia SingaporeUSA Refrigerator 11 + 5 13 + 5 11 + 6 13 + 4 20 Solutions for questions 14 to 17: Air conditioner 9 + 7 12 + 5 10 + 8 12 + 5 23

14. If one observes the values given for the different Music system 8.5 + 9 10 + 6 8 + 4 13 + 4 20 parameters, the values that were varying with Total 49.5 51 47 51 63 production, i.e., value was increasing when production increased and value decreasing when production The cheapest is in Malaysia. Choice (3) decreased are material, labour and operating cost of machines. All the remaining costs, i.e., rent of building, 19. As given in the previous question, the total cost will be consumables, rates and taxes, repair and maintenance highest in India (850 + 900 = 1750) Choice (1) expense and selling and marketing expenses are fixed. Hence, there will be no change in these costs. The total 20. Cost in India = 300 + 500 = 800 fixed cost = 1800 + 600 + 1200 + 8700 + 2100 = 14400 Cost in Thailand =450 + 600 = 1050 The cost/unit for different variable costs is as follows. Difference = 250 × 32.9 = 8225 ` Material = 50 per unit. Duty = 1500 Labour = `20 per unit Required difference = 6725 Choice (4) Operating cost of machine = `30 per unit ` Total = 100 per unit 21. Cost in India with dollar at `40.92 = 550 × 40.92 ` Selling price per unit = 125 per unit ⇒ 2500 ` 14400 22500 Total cost/unit for 2100 units is 100 + Cost in India with dollar at 35 = = 650 2100 35 ` = 107 Choice (2) Cost in Singapore = 900 Required difference = 250 Choice (2) 15. For one product,

Selling price = `125 Solutions for questions 22 to 26: Variable cost = `100

______22. Let us check the possible short routes from A to J. Difference = `25

______Total cost Total distance Now, to avoid loss, the company has to offset the fixed Rs.335 Rs.1135 cost (i.e., 14400) for which it has to produce a total of A B J `1470 1430 km 14400 280 1150km = 576 units. Choice (3) 25 Rs.625 Rs.1225 A D J `1850 1250 km Rs.25km 825km 16. The reduction in selling price per unit = 5% of 125 = 6.25 Rs.850 Rs.575 A F J `1425 1155 km New selling price = 118.75 670km 485km Total fixed costs = `14400 Variable cost per unit = `100 Rs.1225 Rs.445 A G J `1670 1090 km Now the total profit increases with the increase in 675km 485km number of units sold and the maximum profit is Rs.925 Rs.210 obtained when the company sells and 3000 units. A H J `1135 1175 km Choice (5) 975km 200km Rs.675 Rs.215 17. The given condition is that if the company sells upto A C 2100 units, the selling price per unit is `125 and if the 395km 205km `1465 1085 km company sells 2550 units, the selling price per unit for Rs.575 all the units is `120. The profit of the company F J 485km increases upto a production figure of 2100 units, from the 2100th unit to the 2101st unit, the total profit decreases drastically and from the 2101st unit to the The shortest possible route is A – C – F – J. 2550th unit, the profit again increases. The cost is `1465. Choice (4)

Hence, the profit would be maximum at the production 23. The route with the least cost is A – H – J, with a total figure of 2100 units or at 2550 units. cost of `1135. As the cost of the new service is 5% less then `1135, it should be 1135 – (5% of 1135) = 1078. Production 2100 units 2550 units Choice (2) ` ` Selling price / unit (s) 125 120 Variable cost / unit (v) `100 `100 24. If C, D and H are closed, then the minimum cost of S – V `25 `20 travel is for A – F – J, i.e., `1425. Choice (3) 25 × 2100 20 × 2550 (S – V) production = 52500 = 51000 Pr ice 25. We want the to be as minimum as possible. Total fixed cost 14400 14400 distance Total profit 38100 36600 It is less than 1 in only the cases A – H, B – J and

The maximum profit is `38100 Choice (1) 2 C – D. Considering the cases involving the above 27. Percentage of male employees in the production routes. 288 department = × 100 = 45 Choice (2) 640 Taking margin of 10% Route Price / Distance into Account 28. Post graduates in the marketing department = 32 1135 1135 × 10 25 A – H – J Male postgraduates = × 32 = 8 1175 1175 11 100 1470 1470 10 ∴ A – B – J × Female post graduates = 32 – 8 = 24 1430 1430 11 Male non post graduates = 48 – 8 = 40 Required difference = 40 – 24 = 16 Choice (5) 29. Percentage of male post graduates in the marketing 1135 × 10 It will be the least for A – H – J and is 32 1175 11 department = × 100 = 40 Choice (1) 103.2 80 = = .88 Choice (2) 117.5 30. The number of male post graduates in the production department = 144. 26. The cost / kilometer is the least for A – H – J and the ∴ Female post graduates = 352 – 144 = 208 distance is 1175 km. Choice (4) The number of male and female post graduates and male and female employees who are not post Solutions for questions 27 to 30: graduates are as follows.

With the given information we can deduce the number of males and postgraduates in the different departments as follows: Post graduate Non Post graduates Male Females Males Females Department Total Male Post graduates 144 208 144 144 Marketing 80 48 32 Accounts 80 44 40 Production 640 288 352 It can be seen that except female post graduates all Total 800 380 424 other groups (male posts graduates, male and female non post graduates) have the same number of employees. Choice (3)

Solutions for questions 31 to 33:

31. The total number of bookings made is the highest in Q3 and so the average number of bookings per month is also the highest. Choice (C)

Month Jan Feb March April May June July Aug Sep Oct Nov Dec Number of bookings 346 412 380 450 308 359 462 333 345 250 506 370 216 160 225 170 159 296 134 50 125 278 Number of deliveries 200 (146) (196) (220) (225) (138) (200) (166) (199) (295) (125)

32. The values shown in the brackets are of the booking 36. The given condition is satisfied in the case, where the made 2 months ago. ‘number of wins’ is in the range 25 – 27 i.e., Number of deliveries made in August from the bookings 68 + 64 + 63 = 65 = 2.5 × 26 Choice (A) made in June = 200. 3 Number of deliveries made in December from the

bookings made in November = 278 Solutions for questions 37 to 39: 278 = 1.39. Choice (A) 200 37. Investment (in `) in NLP Industries before withdrawal = 12.5% (12,00,000) = 1,50,000 33. We only need to check the revenue for quarters Q3 and Investment (in `) in NLP Industries after withdrawal Q4. = 16% (9,00,000) = 1,44,000 Revenue (in `) from Q3 = (462 + 333 + 345) × 43,100 ∴ the percentage change in investment = 4,91,34,000 1,50,000 −1,44,000 ` × = ×100 = 4% Choice (D) Revenue (in ) from Q4 = (250 + 506 + 370) 44,000 1,50,000 = 4,95,44,000

∴ the highest revenue is obtained from Q4 i.e., 38. The return on investment for Mr. Anil `4,95,44,000 Choice (B) 2 25 = × × 7,00,000 = 3500 Solutions for questions 34 to 36: 100 100 The return on investment for Ms. Shivani 34. The given condition occurs in the case where the 2.5 10 = × × 13,00,000 = 3250 ‘number of wins’ is in the range 16 – 18. Choice (B) 100 100

Therefore the required difference = (3500 – 3250) 35. From the given table, the 3rd least percentage occurs in the = `250 Choice (A) 99 − 94 100 last row i.e., for 31–33, which is × 100 = 95 19 39. The three persons A, B, and C made an investment of 5 `10 lakh, `20 lakh and `21 lakh respectively such that = 5 % Choice (C) 9 their investments fall under the schemes X, Y, Z respectively.

3 Their combined return on investment (in ` ) The franchise in Bengaluru will earn more revenue then = 2% (10,00,000) + 2.5% (20,00,000) + 3% (21,00,000) the establishment fees (in each of the two centres) after = 20,000 + 50,000 + 63,000 = 1,33,000 one year. Choice (D) Their combined return on investment after the firm increased the rate of return 44. If a customer spends on an average `300 and `130 at a = 2.2% (10,00,000) + 3% (20,00,000) + 3.3% (21,00,000) Foodie restaurant in class A center and class B center = 22,000 + 60,000 + 69,300 = 1,51,300 respectively, then the total number of customers who ∴ The required increase (in `) are required to come such the revenues are not less = 1,51,300 – 1,33,000 = 18,300 Choice (D) than the establishment fees would be the i.e., 132 × 105 104 × 105 Solutions for questions 40 and 41: + = 44,000 + 80,000 = 1,24,000 300 130

40. Percentage contribution of mono speaker of the Choice (B)

1000 company NOSY in 2001 = ×100 =12.3% Solutions for questions 45 and 46: 8100 Percentage contribution of mono speaker of the 45. As no information is given regarding the percentage of 1600 dropouts for districts R and S in few years, (A) cannot company BOSS in 2001 = ×100 =17% 9400 be definitely a false statement. As no information is Percentage contribution of mono speaker of the given about the number of enrolments in each districts companies NOSY and BOSS in 2003 are 13.5% and in any of the years, statements (B) and (D) cannot be 19.6% respectively. confirmed. ∴ the percentage contribution of mono speakers of (C) is definitely false because the dropout percentage of both the companies increased. district Q in any of the given years is greater than that of Proceeding, similarly we observe that for no other type each of the other districts. So, the overall dropout of music systems of both the companies, the percentage would also be the highest. Choice (C) percentage contribution increases. Choice (B) 46. 41. District Minimum number of achievements Percentage Percentage P 5 Type of music system contribution contribution Q 1 in 2001 in 2003 R 2 Mono speaker 12.3% 13.54% S 0 Dual speaker – 1000w 22.22% 22.9% T 2 Dual speaker – 2000w 28.4% 18.75% ∴ Four speakers – 5000w 17.3% 23.95% total number of achievements (minimum = 5 + 1 + 2 Home theatre 19.75% 20.8 + 0 + 2 = 10) Choice (B)

The maximum change in percentage points occurs for Solutions for questions 47 and 48: Dual speaker – 2000W. Choice (C) 46 − 25 47. The rise in temperature (in °C) per hour = = 7 Solution for question 42: 7 ∴ temperature (in °C) in city Q at 10 a.m. 42. The total number of cars sold by showroom A and = 3 × (10 – 5) + 25 = 40 Choice (D) showroom B at the end of 7 days are 209 and 221 respectively. 48. Temperature in city P at 3.30 p.m. 209 × ≈ Ι  42 − 29  1 = 100 94.5%. Hence statement is true. = 42 –   × 3 221  6  2 The total number of cars sold by showrooms on odd 91 numbered days = 16 + 35 + 33 + 51 + 60 = 195 = 42 – ≅ 34.5°C The total number of cars sold by showroom B on even 12 numbered days = 19 + 42 + 29 + 52 + 81 = 223. 90%(223) = 200.7 Similarly, City Temperature at 3.30 p.m. ∴ Statement ΙΙ is also true. Choice (C) Q 35.5 °C R 37 °C Solutions for questions 43 and 44: S 38.5 °C T 36.66 °C 126 × 105 U 33.5 °C 43. Hyderabad: = 52,500 > 51,860; 240 ∴ city S has the highest temperature at 3.30 p.m. 75 × 105 Choice (C) < 42,500 180 ∴ a restaurant in class B center but not class A center Solutions for questions 49 to 52: will earn more revenue then the establishment fee in one year. P Q R S T U Traffic flowing from 3,346 3,752 2,536 2,620 2,952 3,060 144 × 105 Bengaluru: = 60,000 < 60,200 Traffic flowing to 3,504 2,612 3,308 2,852 3,050 2,940 240 90 × 105 49. Total traffic through the route P – Q = 964 + 846 = 1810. = 50,000 < 50,246 Similarly verifying it is easy to see that the maximum 180 traffic flow occurs through the road connecting PQ. Choice (A) 4 50. Looking at the table and relating the diagonal elements, Solutions for questions 59 and 60: it is easy to see that the 2nd least traffic flow occurs through the road connecting Q – S. Choice (D) 59. The gain from the shares of company IV in 2006 was = [448 + 432 − 2(456)] 132 + 51. From the above table, traffic flowing from city Q is the 2 greatest i.e., 3752 vehicles. Choice (B) = 132 – 16 = 116 Choice (1) 52. From the above table, the difference in traffic flow is the least for city T i.e., 3050 – 2952 = 98. Choice (C) 60. We can only evaluate the return from the shares of company III in the years 2002 to 2009. The returns Solutions for questions 53 and 54: were as follows:

53. Interest amount for Mr. A (in `) = 3,600 × 2.2 = 7,920 Year 2002 2003 2004 2005 2006 2007 2008 2009 Interest amount for Mr. B (in `) = 3,800 × 3.6 = 13,680 Gain 122.5 158.5 155 150 170 148 172 195 ∴ The required difference (in `) = 13,680 – 7,920 = 5,760 Choice (B) The highest percentage increase is from 2002 to 2003 and it is 29.38% Choice (2) 1 54. The required average = [5.6 × 4,800 + 6.4 × 4,000] 2 Solutions for question 61:

1 = [26,880 + 25,600] 61. To find the median, arrange the per capita incomes in 2 descending (or ascending) or order. = `26,240. Choice (C)

Per capita income ($) Country Solutions for questions 55 and 56: 24,369 Switzerland 55. Let the total number of employees in company X, 24,337 Germany company Y and company Z be x, y and z respectively. 23,484 United states Male employees in company x who owns both four wheeler and two wheeler = 0.7x × 0.15 (Q 45 + 65 + 5 – 19,207 United kingdom 100 = 15) = 105x 15,350 New Zealand Female employees in company y who owns both four 13,746 Swedes wheeler and two wheeler = 0.3x × 0.1 = 0.03x. 13,477 France ∴ The total number of employees in company X who 11,692 Spain owns both four wheeler and two wheeler = (0.105 + 0.03)x = 0.135x 10,372 Hong Kong percentage of employees = 13.5% 5,663 Brazil Similarly for company Y, the required percentage is 26% 4,965 Latherier Similarly for company Z, the required percentage is 6%. 3,523 Mexico Choice (B) 2,916 Romaine 56. Let the number of employees in either of the companies be ‘n’. Out of the 13 countries, the median is the country The number of male employees in companies Y who satisfy placed 7th. 1 France with a per -capita income of $13,477. the given condition = [100 – (30 + 20)]n = 0.5n 100 40% of 13,477 = $ 5390.8. There are 10 whose per capita income is more than similarly the required number of employees in company $5390.8. Choice (2) 1 Z = [100 – (20 + 10)]n = 0.7n. 100 Solutions for question 62: 0.5n + 0.7n ∴ the required percentage = × 100 = 60%. 2n 62. Given, intra-state services accounted for 60% of total Choice (D) revenues. 2880 ∴ Total revenues = = `4800 crore. Solutions for questions 57 and 58: 0.60

Average selling Total revenues from non A/C general category in Model No. of Bikes Sold 2008 price (in `) 2007 intra-services is given to be 50% of revenues from intra- state services. RL-100 19,500 40,000 45,000 ∴ Revenues from non A/C general category in intra- BCZ 37,500 25,000 28,000 state services = 50% of 2880 Thunder 30,000 31,000 35,000 = `1440 crore Choice (2) WB-150 45,000 20,000 23,000 Muzzle 18,000 52,000 55,000 Solutions for question 63:

57. From the above table, the percentage increase in the 63. The total production of the top four coal producing average selling price is the highest for WB-150. countries is 2536.7 + 1039.2 + 478.2 + 393.9 = 4448 mt 23,000 − 20,000 × 100 = 15% Choice (C) The total production of the bottom four coal producing 20,000 countries is 76.7 + 76.6 + 145.8 + 174.9 = 474 mt. 474 58. The required average (in `) The required percentage = × 100 = 10.66% 4448 40 + 25 + 31 + 20 + 52 = × 103 Choice (1) 5 168 = × 103 = 33,600 Choice (B) 5 5 Solutions for questions 64 and 65: 71. The growth rate in imports of the 4 companies from 2002 – 03 to 2003 – 04 are as follows: 64. Male students who were eligible for selection were A, F, 6.53 Rahual & co: ×100 = 127% G and N and the female students who were eligible for 5.14 selection were L and O 4.7 Therefore the required ratio is 2 : 1 Choice (1) Chandu & co: ×100 = 40.5% 11.61 65. x = 6 and y = 4 0.67 Shiva & co: ×100 = 7.7% There fore 2x = 3y is in the correct choice. Choice (4) 8.72

4.05 Solutions for question 66 and 67: Kanta & co: ×100 = 54.3% 7.46 The Total costs, Operating Expenses, Revenue and the Hence the growth rate of imports is the least for Profitability of the company in the five years are given in the Shiva & co. Choice (3) following table. 72. Trade deficit = imports – exports Total cost (in Operating Revenue Profitability The trade deficit of the 4 companies in 2004 – 05 are as Year `) Expense (in `) (in `) (in `) follows 2009 92200 24050 104200 0.2308 Rahul & co: 2.11 2008 83700 21775 96600 0.2254 Shiva & co: 0.9 2007 89600 22000 112400 0.1957 Chandu & co: 1.07 2006 96600 24730 128200 0.1929 Kanta & co: 0.6 2005 104000 26580 130600 0.2035 Rahul and co has the highest trade deficit in 2004 - 2005 Choice (1) 66. The profitability of the company was the least in the year 2006 Choice (2) Solutions for questions 73 and 74:

67. With respect to the previous year in the years 2006, 73. The number of employees who did not cross the cut off 2007 and 2008 were 7.11%, 7.24% and 6.58% for all the 5 companies are as follows. respectively. In 2009 the total cost increased when compared to 2008. Therefore the maximum decrease No. of employees who did not cross the cut off. was in 2007 and it was 7.24%. Choice (2) A 120 B 225 Solutions for questions 68 and 69: C 100 D 200 Score E 275 Score less Score from Total no of Sections greater than than 45 45 to 85 students 85 Hence E has rejected the maximum number of A 28 72 24 124 employees. Choice (4) B 15 68 36 119 C 18 52 28 98 74. The number of employees who got more than 90% for D 29 58 47 134 the 5 companies are as follows: E 30 60 35 125 Greater than Total 120 310 170 600 Cut off cleared 90% marks 68. Percentage of the total number of students getting A 30 180 B 36 225 120 scores less than 45 = × 100 = 20% Choice (4) C 30 150 600 D 96 400 E 115 300 69. For the sections A, B, C, D and E, the maximum Total 275 1255 number of students getting 48 or more in the

examination was 96, 104, 80, 105 and 95 respectively. 307 Thus the highest among the above values is 105. The required percentage is = 24.5% ≈ 24% Therefore the maximum number of students from a 1255 section who passed in the examination was 105. Choice (3) Choice (3) 75. The sales of a company = The no .of units produced – Solutions for questions 70 to 72: the closing stock. The sales of all the 5 companies in 2009 are as follows 70. The growth in exports of the 4 companies from 2003 - 04 to 2004 - 05 are as follows.. Company Sales 3.15 P 10515 Rahual & co: ×100 = 25.9% 12.15 Q 14310 R 9225 2.1 Chandu & co: ×100 = 14.9% S 7755 14.1 T 11135 2.8 Shiva & co: ×100 = 30.1% 9.3 Hence S had the least sales in 2009. Choice (2) 3.32 Kanta & co: ×100 = 31.9% 76. The sales of R in 2008 = 9000 – 675 = 8325 10.41 The sales of R in 2009 = 10000 – 775 = 9225 Hence Kanta & co has the highest growth in exports. ∴R had lower sales in 2008. Choice (1) Choice (4) 6 Solutions for question 77 and 78: Solutions for questions 84 and 85:

77. 84. The yield return of R in the years are as follows:

Expenses + Yield Return Total income Savings Family: Overhead 2003 1557.14 (in `) (in `) (In `) 2004 1574.1 Kapoor 147000 12000 135000 2005 1884.6 Khanna 105000 13500 91500 2006 2021.7 Kirsten 168000 17750 150250 2007 1595.2 Kumble 140000 16750 123250 2008 1525 Khan 165000 19000 146000 Kittu 120000 17450 102550 Hence the highest yield return is in 2021 Kala 196000 21375 174625 Choice (1) 923175 85. The yield return for Q in the years are as follows The total savings made by all the families was `923175 Choice (2) Yield Return 2003 1676.5 ` 78. The increase in income of the Khan family is 3300 2004 2204.5 ` The decrease in expenses is 570 2005 2633.3 ∴ ` The increase in saving is 3300 + 570 = 3870 2006 2000 Choice (3) 2007 2720 2008 4105.3 Solution for question 79: Hence the highest percentage increase is in 2008 79. The healthy drinks are S are X Choice (4) The other drinks are unhealthy. Hence the required ratio is 2 : 8 = 1 : 4 Choice (3) Solutions for questions 86 and 87:

Solutions for 80 and 81: 86. From the table we can easily observe that the average

marks are the highest for ΙΙ, ΙX and X. 80. The difference in the number of students studying in Hence these classes would satisfy the statement government schools in all the states in 2008 are as "the higher the average marks, the higher are the follows. number of students". Choice (3)

Difference 87. The statement “the lower the number of students, the AP 1800 higher the average marks” can be verified through the MP –1600 options. UP 1300 Classes Ι and ΙΙ have higher number of students, hence Karnataka –1400 Kerala 2200 they do not satisfy the statement. ∴ Tamil Nadu 5200 The correct choice is (A) Choice (1)

The maximum increase is for Tamil Nadu, Solutions for question 88: Choice (4) 88. The number of students this year in the 6 states 81. We can observe that the state which has consistent increase in the number of students from 2007 to 2009 is Number of students UP. Choice (2) AP 13,21,000 UP 17,46,000 Solutions for questions 82 and 83: MP 13,90,000 Bihar 19,14,000 82. The median of the total number of students is Assam 12,88,000 18 + 17 Orissa 10,88,000 =17.5 2 Hence A, C and F have more number of students than Hence MP has the third highest number of students the median. Choice (2) this year Choice (2)

Solutions for question 89: 83. The number of failed students in each section is given the table. Hence the number of failed students is the highest more 89. Given Lakshmi spends 20% of the revenue earned in section C from each investment to maintain her house. So let us calculate the revenue for each business in which she invested. Section No. of students failed

A 6 X Y Z B 8 Investment 16.2 14.5 12.9 C 12 Revenue 16.96 14.72 13.2 D 7

E 3 Hence the maximum profit is obtained from X F 10 Choice (1)

Choice (3) 7 Solutions for questions 90 and 91: 50 − 32 = ×100 = 35 (approximately) Choice (3) 50 90. The income and expenditure for the four regions in

2007 are as follows. Solutions for questions 5 and 6:

Income Expenditure Ratio 100 5. Number of pythons in the world = × 4800 = 12,000 North 33.8 34.5 0.98 40 100 South 33.8 35.2 0.96 Number of bears in the world = × 4200 = 10,000 42 East 31.9 32.7 0.975 The number of deers and wild bisons in South America West 40.1 41.3 0.971 are 6,000 (25% of 24,000) and 5,400 (30% of 18,000) respectively. ∴ Number of wolves in South America Hence the required ratio is the highest for the North = 25,800 – (4,800 + 6,000 + 5,400 + 4,200) = 5,400 region. Choice (1) ∴ total number of wolves in the Amazon forest = 75%

of 5,400 = 4,050. Choice (B) 91. The states in which the per capita income increased by

more than 5% are J and K, West Bengal, Gujarat and 6. Maharashtra

In the remaining states the per capita income did not increase by more than 5% Species Number in Amazon Forest Hence the required ratio is 1 : 1 Choice (2) Pythons 80% (4,800) = 3,840 Deers 70% (6,000) = 4,200 BAR GRAPH Wild Bison’s 80% (5,400) = 4,320

Wolves 75% (5,400) = 4,050 Solutions for questions 1 to 4: Bears 95% (4,200) = 3,990 1. The percentage growth rate in 2007 over 2006 250 − 190 Choice (B) = ×100 = 31.5% 190 Solutions for questions 7 to 9: Had the percentage growth from 2007 to 2008 been 31.5%, the estimated revenue would have been 7. The percentage increase is maximum in case of × 131.5 = 5400 − 4480 250 329 company R i.e., ×100 ≈ 20.5% 100 4480 The required difference 329 – 305 = 25 (approximately) Choice (C) Choice (1) 8. Since the cost of PC is same for all the companies 2. Let the number of people who used the company's market share of Q in 2009 products in Asia in 2003 be 100. 4200 = ≈ 21.76% The number of men and women who used the product + + + in the different years are 5600 4200 5000 4500 Market share of Q in 2014 Year 2003 2004 2005 2006 2007 2008 2009 2010 110%(4200) = × 100 = 28.37% Men 60 63 66.15 69.5 73 76.65 80.584.5 110% (5600 + 4200 +500) Women 40 44 48.4 53.25 58.5 64.5 71 78 ∴ the difference in percentage points = 28.37 – 21.76 Total 100 162.5 = 6.61. Choice (A)

∴ the approximate percentage growth = 62 9. Looking at the options it is enough if we check for the Choice (1) market share of S in 2006 and 2007. 3. The percentage change in the gap between the revenues Let the price per PC of A, B, C and D `x, `2x, `x and from the US and Asia in the different years are `2x respectively. Market share of S in 2006 Year 2003 2004 2005 2006 2007 2008 2009 2010 2 × 5800 Gap in = ×100 150 170 150 140 110 85 60 50 4200 + 3000 × 2 + 448 + 5800 × 2) million USD 116 Absolute = ×100 ≈ 44% percentage 13 12 6 21 22 30 17 42 + 60 + 44.8 + 116 change Market share of S in 2007 54 × 2 108 The absolute value of the percentage change in the = ×100 = ×100 43.5 + 28 ×2+ 48.5 +54× 2 256 growth rate was the highest in 2008-09. Choice (4) = 42.2% Choice (A) 4. The growth rate in 2005 (over 2004) 10. Let us consider the selling prices of the four models in 135 − 90 = ×100 = 50% 2004 as 3k, 4k, 5k and 6k respectively. 90 Selling prices in the years. The growth rate in 2007 (over 2006) 250 −190 Models 2004 2005 2006 = = 32% 190 P 3k 4.5k 6k The required percentage Q 4k 6kl 8k R 5k 7.5k 10k S 6k 9k 12k 8 Sales revenue of Q in 2004 = 750 (4k) = 3000k Therefore the sales revenue of Q in 2004 by Sales revenue of R in 2006 = 500(10k) = 5000k 5000k − 3000k 2 ×100 = 66 % Choice (3) 3000k 3

Solutions for questions 11 to 13:

Car 2007 2008 2009 Production Sales Production Sales Production Sales Alto 13000 8000 15000 10000 14000 9000 Swift 22000 20000 21000 18000 25000 22000 Estio 20000 18000 21000 20000 22000 16000

11. The total production of all three Cars in 4. The student showed the highest percentage increase in 2007 = 55000 84 − 51 QA section i.e., ×100 = 64.7% Choice (B) 2008 = 57000 51 2009 = 61000

The total sales of all three Cars in 5. 2007 = 46000

2008 = 48000 2009 = 47000 Section Percentage change The required ratio in 56 − 34 QA × 100 > 50% 2007 = 1.19 34 2008 = 1.18 55.2 − 33.6 2009 = 1.29 LR × 100 ≃ 40% Hence the ratio is the highest in 2009 Choice (3) 55.2 36.4 − 24 VA × 100 > 50% 12. The ratio of production to sales of Alto in 24 2007 = 1.625 56 − 36 2008 = 1.5 RC × 100 > 50% 2009 = 1.55 36 The required ratio is the highest in 2007 62.5 − 52.4 Choice (1) DI × 100 ≃ 16% 62.5

13. The exports of Swift in 2007 = 2000 Choice (D) 2008 = 3000 2009 = 3000 6. The marks obtained by the student in the RC section is The ratio of exports to sales in the highest in AIMCAT 5 Choice (B) 2007 = 0.1 2008 = 0.17 Solutions for question 7: 2009 = 0.14 The required ratio is the highest in 2008 7. Given the total number of students = 1000 Choice (2) Dancing = 45% = 450 Embroidery Classes = 5% = 50 PIE CHART Singing = 20% = 200 Karate = 15% = 150 Solutions for questions 1 to 6: Painting = 15% = 150 Now only Boys chose Karate. Hence a total 150 Section students in Karate are all boys. QA LR VA RC DI Total Exam Only girls chose Embroidery classes. Hence a total 50 AIMCAT 1 51 69 60 45 75 300 students in Embroidery are all girls. Also 80% of students in Singing are girls. AIMCAT 2 105 30.8 35 56 53.2 280 Hence 160 students in Singing are girls and 40 are AIMCAT 3 120 45 60 54 81 360 boys. AIMCAT 4 80 64 32 64 80 320 Similarly 80% of students in Dancing are boys. AIMCAT 5 84 42 91 70 63 350 Hence 360 are boys and 90 are girls, who are in dancing QA LR VA RC DI In Painting the ratio of boys to girls is 1 : 1 Maximum actual score 100 80 50 60 100 Let us tabulate the data.

60 Boys Girls Total 1. The required percentages = ×100 =120% 50 Dancing 360 90 450 Choice (D) Singing 40 160 200 Painting 75 75 150 2. Maximum possible ‘actual score’ in RC section = 60. Karate 150 0 150 The least difference occurs in the AIMCATs 2 and 4 i.e., Embroidery Class 0 50 50 60 – 56 = 64 – 60 = 4 Choice (A) Total 625 375 1000

3. From the above tables, the given condition is satisfied If Painting & Singing are mixed then the ratio of boys to in AIMCAT 3 and AIMCAT 5. Choice (B) girls is 115 : 235 = 23 : 47 Choice (3)

9 Solutions for question 8: LINE GRAPH

8. Given the ratio of the number of employees in central to Solutions for questions 1 and 2: state government jobs is 6 : 1. Let the central government jobs be 600 and the state 1. Profit on a normal day = 7000 – 6500 = `500 government jobs be 100. Profit when 300 units are sold = 10,500 – 9000 = `1500 The number of central government employees in A.P = 1500 − 500 Required percentage = × 100 = 200% 150. 500 The number of state government employees in Kerala = Choice (C) 25.

∴The required ratio is 150 : 25 = 6 : 1 Choice (1) 2. Cost when 200 units are produced = `6500

Solutions for 9 and 10: Cost when 350 units are produced ≃ `10,000 10,000 − 6500 3500 Additional cost /unit = = = `23. 9. The number of employees in each department of P 150 150 and Q. Choice (B)

P Q Solutions for questions 3 and 4: HR 1750 3780 Academic 10500 3060 The energy consumption of Geyser in a week is 7 kWh and Operations 5250 11160 we know the family uses the Geyser for 2 hrs in a day. Hence for 14 hrs in a week the energy consumption is 7 kWh. Total number of employees in both companies in Hence the energy consumption of a Geyser per day is 1 kWh. the HR department = 5530 Now, energy consumption of Refrigerator in a week is 5530 14 kWh and the family uses Refrigerator throughout the day. Hence the required percentage = = 15.5% 35500 Hence, the energy consumption of Refrigeration per day is Choice (2) 2 kWh. Similarly the energy consumption for TV in a day is 2 kWh. 10. The number of Academic employees = 13560 The energy consumption for Washing machine in a day is The number of Management department employees 4 1 kWh and for Grinder is kWh = 21940 7 7 13560 The required ratio = = 0.618 ≈ 0.62 21940 3. (a) Energy consumed by TV for 3 days = 6 kWh. Choice (2) Energy consumed by Refrigerator for 3 days = 6 kWh. Pie Charts + Bar Charts Hence Choice (1) is false. (b) Energy consumed by Geyser for 4 days = 4 kWh. Solutions for questions 1 and 2: Energy consumed by Grinder for 7 days = 1 kWh. Hence Choice (2) is false. Number of students in each discipline is as follows: (c) Energy consumed by Washing Machine in a week = 4 kWh. Number of students Energy consumed by Geyser for 2 weeks = 14 kWh. Hence Choice (3) is true. Discipline Number of students (d) Energy consumed by TV for 2 days = 4 kWh. Energy consumed by Washing machine for one Marketing 3780 week = 4 kWh Finance 1260 Hence (d) is false Choice (3) Operations 1008 Systems 1512 4. The fixed cost increased by 25 %. Hence the new fixed 1 HR 252 cost is `60 + (60) = `75. 4 Number of males and females in each discipline are as Hence the increment in the total cost is follows: 15 15 ×100 = ×100 = 12.5% 4 120 60 + (0.35 × × 30) Males Females Difference 7 Marketing 2079 1701 378 Choice (2) Finance 819 441 378 HR 1008 1512 504 DATA SUFFICIENCY Systems 840 672 168 Operations 576 672 144 Solutions for questions 1 to 4: Total 5322 4758 1. From A, as 60% of the newly joined employees were 1. The total number of female students in the institute was not managers, the remaining 40% of the newly joined less than the total number of male students by employees were managers. It is given that 5322− 4758 10 managers had newly joined. × 100 = 10.6% Choice (3) ∴ ⇒ 5322 40% = 10 100% = 25 Hence, A alone is sufficient.

B gives no data, it is just an assumption. Choice (1) 2. The difference between the number of male and female

students was the highest for HR. Choice (1) 2. From A and the given condition, either Babu or David

got the highest rank.

Hence, A alone is not sufficient.

10 From B and the given condition, either David or Amar Again yz = 21 the different possibilities are can be the highest ranker. Hence, B alone is not 1 × 21 sufficient. 3 × 7 Combining A and B, David must get the highest rank. But we do not know if x , y, z are natural numbers or Choice (4) not. For eg if y = 9, we can get. 7 3. It is given that, 30% of the students are boys, which xy = 18 as 2×9 and yz = 21 as 9 × 3 implies 70% of the students are girls. Also 10% of the girls are athletes. ⇒ 10% (70%) = 7% of the students There will be infinite possibilities like this, so statement are female athletes. A is not sufficient. From A, 25% of the students are athletes Statement B above is also not sufficient as it gives Hence 25 − 7 = 18% of the students are boys who are information regarding x and z only and nothing about y. athletes. So, A alone is sufficient. Combining both the statements, we can conclude that From B, x = 6, y = 3 and z = 7. Thus z is the maximum. Number of boys who are athletes = 120% of the girls who are athletes. 2. Using statement A alone we have SEVEN = 19 and As 7% of the students are girls who are athletes, 120% FIVE = 14, without knowing anything about the values (7%) = 8.4% of the students are boys who are athletes. of individual alphabets, we cannot answer the question. So, B alone is also sufficient. Choice (3) Using both the statements together we can conclude that 2 (N) + I + E = 7 ⇒ 4. Clearly, A alone is not sufficient, as we do not know N = 1 and I and E are 2and 3 or 3 and 2 how many points the opponent scored. So F + V = 14 – 5 9. F and V could be 4, 5 or 5,4. B alone is also not sufficient, as we do not know how SEVEN = 19 many points team A scored. If E = 2 and V = 5, we get S = 9 whereas if E – 3 and Combing A and B, V = 5, we get S = 7. If the score at the half time was say 0-25, then the So we cannot determine S uniquely. match would have ended in a tie at 35-35. So, team A Thus the question cannot be answered even by did not win. Had the score at half time been, say, 10-35, combining both the statements. then in the end it would have been 45-35 and team A would have won. So, we cannot answer the question 3. The number of days that Raju's dad goes to the Shiva even after combining both the statements. 365  temple in a year is   = 121 days. Choice (5)  3 

The number of days that Raju's dad goes to the PPL 365  Venkateshwara temple is   = 91 days. 1. Let the cost of the new car be `x.  4  2 The number of days that Raju's dad goes to the Therefore the cost of the old car = 40% (x) = x. 5 365  Saibaba temple is   = 52 days. From statement Ι, we cannot answer the question as  7  neither the cost of the new car nor the cost of the old The number of days that Raju's dad goes to both the car is given. Ι 365  Statement alone is not sufficient. Shiva and the Venkateshwara temple is   From statement ΙΙ, we only know regarding his personal  12  saving but nothing about the cost of the cars. = 30 days. Statement ΙΙ alone is not sufficient. The number of days that Raju's dad goes to both the By combining both the statements, we have the 365  following information. Venkateshwara and the Saibaba temple is    28   2  Amount borrowed from his friend = 60%  x = 13 days.  5  The number of days that Raju's dad goes to both the 3  2  6 365  =  x = x Shiva and the Saibaba temple is   = 17 days. 5  5  25  21  2 The number of days he goes to all the three temples is Money realised by selling the old car = x 5 365    = 4 days. Money withdrawn from personal savings account to  84  meet the cost of the new car. Hence the number of days Raju's dad goes to exactly 6 2 9 = x – x – x= x one temple is 121 + 91 + 52 – 30 – 13 – 17 – 4 25 5 25 = 204 days. Choice (1) 9 Now, it is not known that x was what portion of his 25 4. A + B +C + C + D + E +E +F + G = A + B + C + D + E +F +G + C + E personal savings balance. (1 + 2 + 3 + 4 + 5 + 6 + 7) + C +E Thus the question cannot be answered even by Now 28 + C +E = 33. combining both the statements. Choice (4) ∴C + E = 5

C, E could be (1, 4), (4, 1) (2, 3) OR (3, 2)there are four (Numbers) possible ordered pairs of (C, E). Choice (3)

1. From statement A,

xy = 18 the different possibilities are:

1×18 × 2 9 × 3 6 11 CASELET Solutions for questions 8 to 12:

Solutions for questions 1 to 3: The trading pattern followed by each of the three traders is as follows The arrangement of the buildings according to the given conditions is Anand Bala Chandu Yellow Blue Indigo Buy Sell Buy Sell Buy Sell C B E 10 a.m. 3 p.m. 10 a.m., 3 p.m. 10 a.m., 3 p.m. 11 a.m., 11 a.m., 12 noon, 12 noon,

1 p.m., 1 p.m.,

2 p.m. 2 p.m.

8. As the direction of the price movement is not known, the profits of Bala and Chandu depends upon the prices A F D at which they bought gold i.e., if they buy at lesser price Orange Green Violet Height than that bought by Anand, their profits would be more, 1. E if not, the profits of Anand would be more than that of 2/3. B/D the other two. Hence the answer cannot be determined. 4. A Choice (5) 5. C 6. F 9. Anand buys the entire quantity at a single point of time, whereas each of the other persons buy once every 1. The colour of the building diagonally opposite to the hour. As the direction of movement of gold is not given, yellow coloured building is Violet. Choice (4) we cannot compare the returns of Anand with the other two persons. 2. The second tallest building is either B or D. Bala: Bala buys the same quantity of gold every time, Choice (5) irrespective of the price. 3. The colour of the tallest building is Indigo. Chandu: Chandu spends the same amount every time, Choice (2) his buying depends on the price of gold at the time he buys. The more the price, the lesser quantity he buys. Solutions for questions 4 to 7: As his strategy is based on prices, whenever the prices are changing, Chandu’s returns will be more than that Stage Ι of Bala. But if there is no change in the price of gold the As P, Q, S and T won at least one match, R and U lost all returns of Bala and Chandu would be equal. Hence no the three matches. conclusion can be made. Choice (5) As Q, S and T lost at least one match, P won all the three matches. 10. On a boom day, the price of gold keeps rising, hence it In stage-Ι, there are a total of 9 matches and so 9 wins. will be the least in the morning. Hence, Anand who Q, S and T won two matches each. bought all his holdings in the morning will get the As P (the top team in stage-Ι) did not play against maximum profit. Between the remaining two, Bala U, P played matches against Q and R. bought the same quantity at every time, i.e he bought ∴The ninth match was between Q and U. the same quantity even at higher prices whereas So the nine matches that have taken place are as follows. Chandu spent the same amount. Hence, Chandu bought less quantity of gold when prices were high and Won Lost Won Lost Won Lost more when prices were less. Hence, Chandu’s returns are more than that of Bala's. Bala will have the least P S S R S U returns. Choice (1) Q T T R T U

P Q P R Q U Let the prices of gold at different timings be as follows.

ΙΙ Stage- Time 10 a.m. 11 a.m. 12 noon 1 p.m. 2 p.m. 3 p.m. ΙΙ As each team played a total of five matches, in stage , the Price a b c d e f matches take place between the following pairs of teams. P – T, P – U, Q – R, Q – S, T – S and R – U We will look at the additional information given: ΙΙ Given that, in stage- , three teams lost all the two matches. The quantity bought by Anand at 10 a.m. is the same as the ΙΙ Given P lost both the matches in stage- quantity he sold at 3 p.m. As it is given that Anand lost money, ∴Each of T and U won the two matches. we can ignore the quantity bought/sold and can conclude that ⇒ R and S lost the two matches. the price at 3 p.m. must be less than that at 10 a.m. ∴Q also won two matches. ⇒ a > f → (Ι) Similarly the quantity of gold bought/sold by Emma in each 4. T and U defeated P (the top team in stage-Ι) instance is the same and it is given that Emma made a Choice (2) profit. Hence we can conclude that (c + f) > (a + d) → (ΙΙ) Also using similar logic in case of David, we conclude that 5. Only Q, T and U won both their matches in stage-ΙΙ. (d + e + f) > (a + b + c) → (ΙΙΙ) Choice (4) It is given that the price increased from 2 p.m. to 3 p.m. ⇒ e < f → (ΙV) 6. S and U won exactly two matches in the event. It is given that price at 12 noon was lower than the opening Choice (5) price ⇒ c < a → (V) From (i) and (ΙΙ) we can conclude that c > d → (VΙ) 7. Q and T won exactly four matches each in the event. From (Ι), (ΙΙΙ) and (VΙ) we conclude that e > b → (VΙΙ) Choice (5) Hence a > f > e > b and a > c > d ⇒ a is the highest. 12 11. The price of gold was the highest at 10 a.m. Students who opted for Finance = a + e + d + g. Choice (1) (a + e + d + g) 50% = b + f + c. → (1) Students who did not opt for HR = a + e + b. 12. As d < c, choice (4) is also necessarily false. 5 a + e + d + g = (a + e + b). → (2) Choice (4) 4

Again number of students who opted for only Finance Solutions for questions 13 to 15: 1 and Marketing was 33 / % of those who opted for all 3 three 13. The different possibilities in which they could have Number of students who opted for only Marketing and booked the rooms are as follows. Ι HR = Number of students who opted for only Finance Case : and Marketing.

1 101 102 103 104 ∴ f = e = g. 3 A C D B ∴ f = e = k. Case ΙΙ: From equation (1), a + e + d + g = 2 (b+ f + c) a + k + d + 3k = 2 (b + c) + 2k a + d = 2(8k – a) – 2k. 101 102 103 104 3a + d = 14k → (3) B D C A From equation (2), we get, 4d + 11k = 5b + a → (4)

Again number of students who opted for only Finance & Since B booked an odd numbered room, we can HR, i.e., d was 50% of those who opted for only conclude that as per case ΙΙ, B must have booked room Finance i.e., a. number 101, in which case C would have booked room ⇒ d = 50% a. number 103. Choice (C) Substituting in equation (3), we get d = 2k and a = 4k.

Substituting in equation (4) we get b = 3k. 14. It is given that two girls failed in the examination. Now Now (a + b + c + d + e + f + g) = 15k we have six possibilities in which we can select the two ⇒ girls who failed. They are as follows: 15k = 270 k = 18 ∴ Exactly two = d + e + f = 4k = 4 (18) = 72 students. Choice (B) Cases ↓ Dolly Molly Polly Kelly

1 Pass Pass Fail Fail Solutions for questions 16 and 17: 2 Pass Fail Pass Fail 3 Pass Fail Fail Pass It is given that books D and F was read by the same person, 4 Fail Pass Pass Fail A and B was not read by the same person and F and C was 5 Fail Pass Fail Pass not read by the same person. 6 Fail Fail Pass Pass The different combinations in which the books were read are as follows: Let us denote a true statement by T and a false statement by L (lie) Ι ΙΙ ΙΙΙ ΙV A B A B A B A B Cases→ 1 2 3 4 5 6 D C D C D C C D Dolly T T T L L L F G F E F E G F Molly L L L L T L E H G H H G H E Polly T T L L T T Kelly T L T T L T V VΙ A B A B As exactly three of them were telling the truth, only in C D C D Ι case it is so. Thus Molly was the person who was E F E F lying. Choice (B) H G G H

15. 16. As Akira read books E and G, the books that Akira read

Finance Marketing could be either A, C, E and G or B, C, E and G. In either

case, we can conclude that Aroki did not read book C.

a = 4k Choice (D) b = 3k

e = k 17. As books C and E, were not read by the same person,

g=3k as in cases Ι and ΙV, books G and H were read by the

same person. Choice (C) d = 2k f = k

c = k Solutions for questions 18 to 25:

HR

18. W and W are allotted a shift, one earlier than W and n 5 7 6 W and W are also allotted a shift earlier than W . 3 9 6 Again as W is allotted a shift lower than W , if we allot 3 2 the afternoon shift for W and W ; W and W being one It is given that n = O 3 9 5 7 shift earlier than W , we will have four workers in the g = 37.5% of (a + b + c) 6 Afternoon shift if W is allotted the Evening shift. Thus 3 6 i.e., g = (a + b + c) the only shift that can be alloted to W6 is the Night shift. 8 The following table gives the workers and the shift they Let (a + b + c) be 8k. were allotted to. ∴ g = 3k Students who did not opt for Finance = b + f + c 13 Morning W2 Case (3) Day 1 / 2 / 3 Afternoon W1, W3, W9 Schumi Mclaren Sebastian Evening W5, W7 Sebastian Schumi Mclaren Sebastian Schumi Mclaren Night W4, W6

Thus W8 can be allotted any shift other than the If Schumi beats Mclaren on all the three days, then afternoon shift. Choice (B) Mclaren will come last all the three days (not possible). Choice (D) 19. The following table lists down the matches and the corresponding players who led the team as captain and 25. (i) Both Sashi and Govind work together. This implies vice captain. Ryan and Mokambo will work together Choice (A) Match Captain Vice Captain Match 1 B A/D Solutions for questions 26 to 28: Match 2 A C Match 3 B A/D 26. Let the runs scored by Bhajji be x Match 4 C A/B/D Match 5 D A/B/C Straight drive Pull shot Others Total x + 40 As D refused to lead the team as captain if A or B led Pollard x + 40 5 the team as captain in the preceding match, we can conclude that D can be the captain of the team only in Dumminy (0.6) (x+ 20) (0.15) (2x + 40) x + 20 Match 5. x Bhajji = 20 x Again with D as Captain in Match 5, A must have 4 captained the side in Match 2 for A cannot be the captain in Match 4. x Now with A as the captain in Match 2, C must have Given = 20 been the vice captain in that match. 4 Thus C was the vice captain in Match 2. ⇒ x = 80. Choice (C) Straight drive Pull shot Others Total 20. For B: Pollard 24 120 Person Appliances Day Dumminy 60 10 30 100 Water Purifier Monday Bhajji 20 80 B Refrigerator Tuesday AC Wednesday Maximum possible difference = 95 – 10 = 85 Choice (D) For A or C: AC must be bought before the water purifier. 27. Maximum runs scored by Bhajji through straight drive = 59.

Choice (B) 59 2 ∴ Required percentage = × 100 = 19 /3% 300 21. Number of matches played by Sachin is equal to that Choice (D) played by Mongia. Number of matches played by Dravid is equal to that played by Hussey. Since Sachin 28. Runs scored by Bhajji through ‘others’ cannot be has played more matches than Dravid, the average determined. Choice (D) runs must be less than 45. Choice (A) Solutions for questions 29 to 41: 282 22. Average (Ramesh) = = 40.28. 7 29. (1) All shoes are pens. 301 (2) Not all pens are pencils. Average (Sanjay) = = 43. (3) All pens are chocolates. 7 (4) Not all chocolates are pens. If the average score after the exclusion lies between 40.28 and 43, then the average of Ramesh will Analyzing the options: decrease while that of Sanjay will increase. Since, (A) Combining (1) and (4), we get “Some chocolates 92 is the only value lying in that range, so their score in are not shoes”. the invalid question is 42. Choice (A) (B) Combining (1) and (3), we get that ‘some shoes are chocolates’. Choice (D) ⇒ 23. A wins B wins ⇒ B wins C does not win. 30. That implies both A and C do not win together. →t That means at most one of A or C wins. That further implies that D must win. Choice (A) • • Ramesh’s house Umesh’s house ← 24. Case (1) Day 1 / 2 / 3 4 t Schumi Mclaren Sebastian 5 Schumi Mclaren Sebastian 4 9 Total time = t + t = t Sebastian Schumi Mclaren 5 5

9 Case (2) Day 1 / 2 / 3 t = ()7 : 45 − 4 pm = 225 min. Schumi Sebastian Mclaren 5 Schumi Mclaren Sebastian t = 125 min Sebasian Schumi Mclaren 14 When Ramesh reaches Umesh’s house, his watch was Emmanuel’s rank in Physics must have been 3. showing 4 p.m. + 125 mins = 6:05 p.m. Ben got the same rank in Mathematics and Chemistry. Umesh’s watch was showing 6:10 p.m. So, Ramesh’s Remaining ranks of Ben is be 4, 4 and 12. Therefore watch is 5 mins slower than Umesh’s watch. Ben or Emmanuel did not get the 1st rank in Choice (D) Mathematics. Thus Adam’s rank in Mathematics was 1. Cathy got 3rd rank in Mathematics, therefore Ben got 31. the 4th rank in Mathematics. Proceeding like this we can conclude that Cathy got the 1st rank and Adam got the 2nd rank in Physics T1 T2 Choice (B)

34. 200 – x x 300 – x Refrigerators Air Conditioners

y 24 b 20 x 300 − x + y d = a c y 200 − x + y

x Let = 2 26 y LCD TVs ⇒ 400 – 2y = 300 – y y = 100

x If > 2 It is given that at least 40 families own both a y Refrigerator as well as a Air conditioner. then y > 100 and x > 200. This is not possible so only ∴ b + d is at least 40. one value exists Choice (A) We have to find the maximum value of a. a will be maximum when (b + d) is minimum. 32. i.e., when (b + d) is 40. Now a+ c+ (b + d) + 24 + 20 + 26 = 120. Swimming Running Cycling Walking Total a + c = 120 – 70 – (b + d) W 5 6 15 a + c = 10. ∴ The maximum value of a is 10, when c is 0. X 1 6 Thus at most 10 families owes a refrigerator and a LCD Y 5 2 4 14 TV but not an Air Conditioner Choice (A) Z 1 18 35. The lectures and the days on which they deliver the In case of Z: lectures are tabulated in the following figure. A sum total of 18 is possible when two ‘6s’ and one ‘5’ is there in addition to ‘1’. Monday Tuesday Wednesday Thursday Lecturers L1 L4 L2 L1 Swimming Running Cycling Walking Total L3

W 2 5 2 6 15 L4 can deliver the lecture either on Tuesday or on X 1 6 Wednesday. Y 5 2 4 3 14 Now, if L4 delivers his lecture on Wednesday, then L3 Z 6 6 1 5 18 cannot deliver his lecture on any of the given days. [Since L3 delivers a lecture only if L2 delivered a lecture The above table gives the ranks obtained by the four on the preceding day and L3 and L4 do not deliver persons in the four events. lectures on consecutive days.] In case of X, a sum total of 6 is possible only if Thus L4 delivers the lecture on Tuesday. 1 + (1 + 3 + 1) is there. He has to get rank 3 in cycling Now, the only day on which L3 could have delivered the and rank 1 in each of running and walking. lecture was Thursday Choice (D) Choice (A) 36. The different ways in which the committee can be 33. formed is as follows: 1. B2 B4 B5 G2 G3 Name 2. B1 B4 B5 G1 G3 Adam Ben Cathy Dimitry Emmanuel 3. B1 B4 B5 G2 G3 Subjects 4. B1 B3 B5 G1 G3 Mathematics 1 4 3 2 5 5. B1 B3 B5 G2 G3 Physics 2 5 1 4 3 6. B2 B3 B5 G2 G3 Therefore there are six ways in which the committee Chemistry 3/5 4 5/3 1 2 can be formed Choice (D) Biology 5/3 2 3/5 1 4

Total 11 15 12 8 14 37. In order to have the total machining time as minimum, none of the machines must be idle at anytime and the Since the sum of the ranks of Dimitry was 8 and he got total time taken must be 10 hours. (i.e., higher of the the same rank in Chemistry and Biology, his ranks in total machining times in the two machines). Chemistry and Biology was 1. Let us consider the answer options and check if it is Therefore Dimitry’s rank in Mathematics was 2 which possible. was the same as Emmanuel’s rank in Chemistry. Option A 15 Case 1 M belonged to Germany. M1 M2 9

Duration UK Germany France Switzerland Turkey

2 P3 P1 M6 M9 M2 M1 M7. M8 M10 M4 M3 M5 3 P1 P2 Case 2 M9 belonged to Turkey. 5 P2 (4) P3 (5) Total = 10 UK Germany France Switzerland Turkey M6 M5 M2 M1 M7 Option B M8 M10 M4 M3 M9

(As M5 did not belong to France or Switzerland) M1 M2 Thus M10 belonged to Germany Choice (C) Duration 40. It is given that sum of the costs of the gifts bought by 3 P P 1 2 Sneha and Sushma was equal to the cost of the gift 2 P3 P1 bought by Shikha. 5 P (4) P (5) We have four possibilities which satisfies this. 2 3 Sneha Sushma Shikha Sushmita Total = 10 Case 1. 800 1200 2000 2800 Case 2. 1200 800 2000 2800 In case of option (C), if product P2 is machined in M1 Case 3. 800 2000 2800 1200 before Product P1, since P2 takes 4 hours in M1, it can Case 4. 2000 800 2800 1200 be done as follows: Again the difference between the cost of Sushma’s gift Duration 0 – 4 4 – 7 7 – 9 and Sushmita’s gift was equal to the cost of Sneha’s M1 P2 P1 P3 gift. This is satisfied only in Case 3. Thus the cost of the Duration 0 – 5 5 – 7 7 – 10 gift bought was Shikha was `2800 and she bought a M2 P3 P2 P1 pair of shoes. Choice (D) The total time taken is again 10 hours Choice (D) 41. As per the conditions given the different ways in which the terms for the two contest can be selected as 38. follows:

Football Cricket Debate: PVQ PVT PVS PVR PVT PVRPVR PVS PVU PVU PVR Elocution: PURPUR PURPUT PUS PUSPUT PUT PQS PQT PUQ a e b Debate: PVS PVT g d f Elocution: PUQ PUQ Option A is false as can be seen in the following cases: Debate: PVR PVS PVT e Hockey Elocution: PUQ PUQPVQ Option B is true. If V and U are in the same category it must be for Debate. We know that U being in debate implies R is not in elocution. Again since only one Those playing exactly 3 games = g among S and T can be selected for a particular Those playing exactly 1 game = a + b + c category, Q must be selected. Those playing exactly 2 games = d + e + f Option C is true as can be seen in the following cases: Those playing at least 2 games = d + e + f + g Debate: PVR PVS PVT It is given that d + e + f + g = 18 → (1) and Elocution: PUQ PUQPVQ (a + b + c) + (d + e + f + g) = 30 → (2). Thus only statement given in option A is false Therefore a + b + c = 12. Choice (A) Now (a + b + c) = 3 (g). or, 3g = 12 NETWORKS or, g = 4. Therefore d + e + f = 18 – 4 = 14. 1. The cost incurred will be minimum when the distance Thus the number of members playing exactly two travelled is the minimum. games is 14 Choice (B) The distance travelled is minimum when he takes a bus going via A E D F G H. 39. We can list down the names of the countries and the The minimum cost incurred by him = 5 + 8 + 4 (10) athletes belonging to them as follows. = `53 Choice (3)

Countries UK Germany France Switzerland Turkey 2. If the road connecting A to E is under repair, then to 1. M6 M7 incur minimum cost, one must board a bus going via the 2. M8 route A D F G H. Since the total distance travelled along this route is the least. ` It is given that M6 and M8 belonged to UK where as M7 The cost incurred = 5 + 8 + 4 (12) = 61 belonged to Turkey. Choice (3) Now M5 and M9 were from different countries and M9 did not belong to France or Switzerland. QBR (Miscellaneous) So M9 belonged to either Germany or Turkey. Now M1 and M3 belonged to the same country and so 1. Given that the number of people rightly reported is 275. did M2 and M4. This includes people under C3 and C4. Let us consider two cases. ∴C3+ C4 = 275 ...... (1)

16 ∴Number of people wrongly reported = 450 –275 = 175 Solutions for questions 4 and 5:

∴C1+ C2 = 175 ………. (2) Given the number of infected people is 50% that of 4. From the choices only option (a) indicates “low per non-infected capita income and low happiness quotient.”

∴Number of infected people = 150 = C1 + C4 ……. (3) Choice (1) And the number of non-infected people = 300 = C1 + C3 ……. (4) 5. From the choices only option (b) indicates high Required difference is between C2 and C4, obtained happiness quotient and high per capita income”. Choice (2) by (2) – (3) ⇒ C2 – C4 = 175 – 150 = 25 Choice (2)

DI (Miscellaneous) Solution for question 6:

1. Given that for every `2 increase in the selling price 6. per ball, the number of balls sold decreases by 20. ∴ Essay If the selling price of each ball is increased k times, Written WE Interview GD selling price = `59 + 2k. writing ∴Profit per ball = (59 + 2k) – 50 = 9 + 2k. Rahul 5 3 2 3 4 Number of balls sold = 700 – 20k. Ramya 5 1 3 4 3 ∴Profit obtained = (9 + 2k) (700 +20k). Profit = 10 ((9 + 2k) (70 –2k)) = 10 (630 –18k + 140k –4k2) The cumulative score of Rahul is = 10 (630 – (4k2 –122k)) 5 × 0.3 + 3 × 0.1 + 2 × 0.25 + 3 × 0.1 + 4 × 0.25= 3.6 The cumulative score of Ramya is   2  2   −  − 61 −  61  5 × 0.3 + 1× 0.1 + 3 × 0.25 + 4 × 0.1 + 3 × 0.25 = 3.5 = 10630 2k       2   2   The required difference is 0.1 Choice (1)  2 2    61  61  = 10 630 +   − 2k −    2   2   Solutions for 7 and 8:   61 61 7. The expected pay-out for Raju is ∴Profit is maximum when 2k – = 0 ⇒ 2k = × × × 2 2 80 0.5 + 40 0.3 – 20 0.2 = 48 Choice (2)

But since k is an integer, 2k must be an integer. 8. The expected pay out for Ramu is 60 62 ∴2k can be taken to be or 80 × 0.5 + 60 × 0.3 – 20 × 0.2 = 54 2 2 After the change of probability the expected pay out for If 2k = 30, profit = 10 ((9 + 30) (70 – 30)) = 10 × 39 × 40 Ramu is 80 × 0.3 + 60 × 0.5 – 20 × 0.2 = 50 ` = 115600 4 × × ∴The required percentage decrease is ×100 If 2k = 31, profit = 10 ((9 + 31) (70 – 31)) = 10 40 39 54 = `115600 = 7.4% Choice (1) ∴For maximum profit, ` ` Selling price = 59 + 2k = 59 + 30 = 89 or 59 + 31 = 90 Solutions for question 9: When selling price is `89, balls sold = 700 – 300 = 400 ` When selling price is 90, balls sold = 700 – 320 = 380 9. The hotel cost for Ramu = $600 Of the given choices, only (A) satisfies. Choice (1) The cost incurred for city tour = $40 The cost incurred for tour of the Hunters valley = $35 2. Total yield from scheme = 0.25 (–3) + 0.55(80) + Hence the total cost incurred by Ramu = $675 0.2(100) = –7.5 + 44 +20 = 56.5 Choice (3) Therefore the total yield from scheme II was also 56.5. Let the probability of the bearish market be p. 10. Given that there has to be a male in every group. ∴ The probability of the bullish market = 1 – 0.4 – P Hence only three groups can be formed. = 0.6 – P Also given P, S are in same group and each group has Now, p (– 10) + 0.4 (60) + (0.6 – p) 100 = 5605 atleast one JSE and one SSE. 110 p = 60 + 24 – 56.5 As both P and S are JSE. The team should have one SSE. ∴p = 0.25 Choice (3) Given R is in a group of 3 people Hence the three groups should have 3, 3 and 3. Scheme 1 2 members in each. Now considering the condition one JSE and one SSE in Market each group we get the following possibilities. Probability Yield percentage conditions (i) (ii) Bearish 0.2 –30 Group 1: R X Y R X Y Group 2: Z P S Z Q Steady 0.45 80 Group 3: W Q W P S

Bullish 0.35 100 From the above possibilities we can conclude that X should definitely be a member of a group which has 3 The yield from scheme = 0.2 (– 30) + 0.45 (80) + 0.35 people. Choice (1) (100) = – 6 + 36 + 35 = 65 Increase in the total yield from scheme 1 65 − 56.5 = ×100 = 15% Choice (4) 56.5

17 Solutions for questions 11 and 12: 17. By observing we can easily find that R and S have the same visibility index. Choice (3) 11. Let the value of the number in column 'b' and row 'd' be 'x' and that in column 'b' and row 'b' be 'A'. Solutions for question 18: 1 Given x = (A + 26 + x) 3 18. The points of A = 1 × 30 + 2 × 20 + 2 × 10 + 1 × 5 = 95 × × × × 1 The points of B = 2 30 + 1 20 + 1 10 + 3 5 ⇒ 2x = A + 26 ⇒ x = (A + 26) = 105 2 The points of C = 3 × 30 + 2 × 20 + 1 × 10 + 1 × 5 As we know the grid contains only integers = 115 Therefore x should be an integer. The points of E = 1 × 30 + 2 × 20 + 1 × 10 = 80 Hence A should be even. ∴ From the choices only 16 is possible Choice (4) The winner is C Choice (3)

12. Given the numbers in column 'a' are squares of the Quant SI – CI prime numbers, starting with the first odd prime number. Hence the numbers should be 32, 52, 72. 112 and 132. 1. Let the number of years after which his interest in Thus, the required sum is 32 + 52 + 72 +112 + 132 scheme 3 will be more than his interest from scheme = 373 Choice (4) 2 be n. 15 Solutions for questions 13 and 14: Interest from scheme 2, Ι2 = (10000) n = 1500n. 100 Given total number of sarees = 400 2n  10  The ratio of Kanchipattu, Benarasi and Mangalgiri sarees is Interest from scheme 3, Ι3= 10000 1+  – 10000. 5 : 3 : 2  200 

The number of Kanchipattu sarees = 200 For n = 8, Ι3 = `11829 & Ι2 = `12,000 The number of Benarasi sarees = 120 For n = 9, Ι3 = `14066 & Ι2 = `13,500 The number of Mangalgiri sarees = 80 ∴After 9 years, Ι3 > Ι2. Choice (2) Given on day 1 he sells 20% of the total which is 80 on day 2 he sells 200 and on day3 he sells 120. 2. Let us consider the interests received by him from the Also on each day he sells the sarees in the same ratio as he four schemes across the year with `1000 invested in bought i.e 5 : 3 : 2 each scheme. K B M Total Day1 40 24 16 80 Scheme 1 Scheme 2 Scheme 3 Scheme 4 Day2 100 60 40 200 Year 1 80 150 102.5 80 Day3 60 36 24 120 Year 2 166.4 300 216 166.4 Total 200 120 80 400 Year 3 259.7 450 340 259.7 Year 4 360 600 477 360 rd 13. On the 3 day he sold Benarasi saree at `480. Year 5 469 750 629 469 ∴Total amount received by him on the 3rd day is Year 6 587 900 796 587 60 × 350 + 36 × 480 +24 × 375 = `47280 Year 7 714 1050 980 714 Choice (2) Year 8 851 1200 1183 851

14. 25% of the total number of sarees were slightly In scheme 4, amount at the end of the year = 1000 (1.2) damaged. = 1200. 1 Amt. remaining after paying the administrative charges ⇒ (400) = 100 = 0.9 (1200) = 1080 4 Amt. at the end of the second year = 1080(1.2) = 1296 The ratio of the damaged sarees of each type is 5 : 3 : 2. Amt. remaining after paying the administrative charges Hence the number of damaged Kanchipattu sarees = 1296 (0.9) = 1166.4 = 50 This scheme is similar to scheme 1. The number of the damaged Benarasi sarees = 30 Therefore scheme 2 produces the maximum interest at The number of the damaged Mangalgiri sarees = 20 the end of 8 years. Choice (2) He sold all the damaged sarees at 20% loss. ∴ Total amount = 50 × 280 + 30 × 320 + 20 × 300 LA (Venn Diagram) = `29600 Choice (3) 1. Let the number of students who applied for all three Solutions for question 15: examinations be x. The number of students who applied for at least 2 of the 15. The population of China in 2009 = 1.6 billiion 3 examinations = 36 – 2x As it increases by 12% per annum it becomes 5.56 billion in 2020 CAT FMS The population of China in 2020 is 15% of the total 9–x population 5.56 Hence the total population is = 37 billion x 0.15 15–x 12–x Choice (2)

Solutions for questions 16 and 17: XAT

16. The graph gives visibility index of 26 people. The visibility index of 14 people are more than U. It is given that 25% (36 –2x) = x 36 – 2x = 4x 14 Hence the required ratio is ×100 = 53.8 ∽ 54% ⇒ x = 6 26 So the completed venn diagram will be as follows. Choice (2) 18 We have the following information.

CAT FMS The Hindu was read by 64 families. ∴ A + d + e = 55  (1) 18 3 9 The times of India was read by 48 families. 6 ∴ B + d + f =  (2) 6 9 The Telegraph was read by 45 families. ∴ C + e + f = 36  (3) 11 XAT Adding equations (1), (2) and (3), we get A + B + C + 2 (d + e+ f) = 130 Again A + B + C + d + e + f = 90 ∴ d + e + f = 40 Number of students in the class = 18 + 3 + 6 + 9 + 6 + 11 ⇒ A + B + C = 90 – 40 = 50 = 62 Choice (3) Therefore exactly one newspaper was read by 50 families. Choice (3) Solutions for questions 2 and 3: Solutions for questions 5 and 6: Speak in Hindi (480) Speak in English (500) Given GT = 300 → 1 GT 300

a + d + f + g = 60 → 2 e = A B b + d + e + g =120 → 3 H S 240 → b c + e + f + g = 180 4 d 120 → a d = t = a + d = f + g 5 g → f e 60 40 a = f = 0 6 c + f = 30 → 7 c C = 180 Equations 5 and 6 ⇒ d = g C n Equations 6 and 7 ⇒ c =30 Using GT formula a + b + c + d + e + f + g + n = 300 Own a car (400) ⇒ 0 + (b + d + e + g) + 30 + 0 + n = 300 ⇒ n = 60 As per the data provided in the question, Using equations 2, 5 & 6, we get d = g = 30. A + e + d = 360  (1) B + e + f = 380  (2) g 30 5. ×100 = ×100 = 10 % C + d + f = 280  (3) GT 300  d + c = 24 (4) Choice (2) (3) – (4) gives f = 40 Again d + 120 = 180 6. From 4 and c = 30, we get e = 120 ⇒ d = 60 Substituting in 2, Eqn. we get b = 30 ∴ c = 180 Therefore b + n = 90. From (1), A + e = 300 and from (2), e +B = 340. Choice (4) Now A + e + e + B = 400 [ the total number of persons = 800] 7. Given, of the 300 students, 70 choose MS A + e + e + B = A + e + B + e = 300 + 340. Hence 230 choose MBA. 230 ∴ e = 240 Given g = 20 → 1 M ∴ A = 60 and B = 100 n = 0 F a + d + f + g = 100 → 2 d b 2. The number of persons who can speak in both Hindi a b + e + d + g = 150 → 3 g and English = 240 + 120 = 360 e d = 2f = g f 360 The required percentage = × 100 = 72% ⇒ d = 20, f = 10 → 4 c 500 From 1, 3 and 4, we get n = 0 Choice (3) b + e = 110 H GT = a + b + c + d + e + f + g + n 3. The proportion of people in the locality who do not own 230 = 100 + b + e + c B + e + A + d + C a car or cannot speak in English = ⇒ c = 20 Choice (3) 800 640 8. Given the number of children who buy T & J (A) = 16 = = 0.8 Choice (4) 800 The number of children who buy C & H (B) = 26 The number of children who buy B & B (C) = 34 4. A + B + C + D +E +F = 120 – (21 + 9) = 90 We know A + B + C = Ex1 + 2Ex2 + 3Ex3 Where Ex1, Ex2 and Ex3 denote the number of children The Hindu The Times buying exactly one, exactly two and exactly three toys of India respectively Given every child buys exactly 2 toys Hence Ex1 = Ex3 = 0 d A B ⇒ 76 = 2Ex2 ∴Ex2 = 38 9 Hence there are 38 children who visited the shop e f Choice (4)

C Solutions for questions 9 and 10: 21 Given P = 24 ……… (1) The Telegraph Q = 36 ……… (2) 19 R = 29 ……….. (3) P Q Therefore the difference between the highest marks S = 25 ……….. (4) a e b obtained is 20. Choice (1) e + l + o + n = 12 …….. (5) f + l + k + o = 18 ……… (6) 4. The least score is obtained when the person attempts R g + m + l + o = 16 ...... (7) f l g c the following 3 papers. K = 8 and l + o = 6 …….. (8) o m k h From (5) & (8) we get Paper 1 Paper 2 Paper 4 j n i S e + n = 6 d Correct 4 2 4 From (1) & (6) we get P Wrong 0 2 0 a + e + f + l + k + o + j + n = Score 60 10 80 24 a + e + n + j = 6 Total score = 150 Choice (2) but e + n = 6 ⇒ a = j = 0 As the people who like S also like R. Solutions for questions 5 and 6: Hence d = i = n = j = 0. h + m + k + o = 25 Therefore e = 6 and b = 4.[As b + (g + l + m + o)= 36, where The final arrangement of the persons in the 9 seater van (g + l + m + o) = 16] was as follows: From (6) & (8) f = 4 From (7) & (8) g + m = 10 1 2 3 From (3) c + h + g +m + l + o + f + k = 29 U X Q c + g + l + f + 25 = 29 c = g = l = 0 4 5 6 Therefore h = 1, m = 10 and o =6 W T R

7 8 9 9. The number of people who like only Q = 14 Choice (4) S P V

10. The number of people who like all 4 movies is 6 5. After the given swapping the final arrangement will be Choice (2) as follows:

LA (Miscellaneous) 1 2 3

1. Ranking of the stores and the total net scores for the W T Q

pizza stores are as follows. 4 5 6

Ranking as S X U Ranking as Stores per delivery Total net score per price. 7 8 9 time R P V A 3 3 3(0.7) +3(0.3) = 3.0 B 6 2 6 (0.7) +(0.3) = 4.8 C 1 1 1(0.7) + 1 (0.3) = 1.0 Therefore X will be seated beside U D 2 6 2(0.7) + 6(0.3) = 3.2 6. After the given swappings the final arrangement is as E 5 3 5(0.7) + 3 (0.3) = 4.4 follows: F 3 5 3(0.7 + 5(0.3) = 3.6 1 2 3 Therefore pizza store D got the third lowest net score. Choice (2) X V Q 4 5 6 Solutions for Questions 2 to 4: S U R 7 8 9 2. The least score obtained by a person in the four papers can be obtained as follows. T P W

Paper 1 Paper 2 Paper 3 Paper 4 From the choices only "W is in the 9th seat" is correct. Correct 4 3 1 4 Choice (2) Wrong 0 1 3 0 Score 60 45 –20 80 Solutions for questions 7 to 9:

Therefore the minimum net score that the person can 7. The proportion of residents who prefer watching movies get is 165. Choice (4) PA is 0.65 The proportion of residents who prefer surfing the net, 3. PB is 0.68 Paper 1 Paper 3 The proportion of residents who prefer doing both, PA∪B Correct 2 4 is 0.61 Wrong 2 0 The proportion of residents who prefer at least one Score 20 100 between watching movies or surfing net is PA + PB –

P ∩ Total score = 120 A B = 0.65 + 0.68 – 0.61 = 0.72 Paper 2 Paper 4 ∴Proportion of residents who neither watch movies nor Correct 4 1 surf net is 1 – 0.72 = 0.28 Wrong 0 2 Score 110 30 8. Let the population of A be 3k. ⇒ Total score = 140 Population of B is 5k, that of C is 3k and that of D is 4k. 20 ∴No. of residents who prefer chatting with friends Hence the probability of Rajini taking something home in A is 0.36 × 3k = 1.08k 4 C1 = 2 in B is 0.45 × 5k = 2.25k is [since all the baskets are equally likely to 6 C 3 in C is 0.32 × 3k = 0.96k 1 in D is 0.25 × 4k = 1.00k get selected by Rajini] Choice (2) ∴ Highest number is in B Choice (2) Solutions for question 14:

9. No. of residents who prefer chatting with friends was 14. Let us consider P = 1. When there is one goat and one calculated in the previous question. tiger then the tiger eats the goat and gets transformed The average number of residents who prefer surfing net into a goat and stays happily in the forest. 1.9k + 2.75k + 2.04k + 3k = = 2.4275k Now if P = 2 when there are two tigers. Now, if one of 4 the tigers eat the goat then it gets transformed into a ∴ 2 colonies have more than the average number. goat and then the second tiger would kill it. Hence when Choice (3) two tigers are there they would not kill the goat. Let P = 3. When three tigers are there. One of the three 10. The different ways in which oil can be transferred from tigers kills the goat and becomes a goat. Hence the tank B to tank H are remaining tigers would not kill the goat. 1. B E A F D C G H Hence when the tigers are odd numbered then they 2. B E A F G H would kill the goat, else the goat is not eaten by any 3. B E C A F G H tiger. Choice (2) 4. B E C G H 5. B E D A F G H Solutions for questions 15 and 16: 6. B E D C A F G H 7. B E D C G HS Given a + g = 12 Thus there are seven possibilities in all. b + g = 8 GT Choices (c) T P 11. Based on the conditions given in the question, we get b the following possibilities. a g

Arun Varun Kiranmala n 1. India-Day-to-Day India-Every-day India-These-days 2. India-These-Days India-Day-to-Day India-Every-day 15. g = 4 The number of days that Ram learnt an instrument is We can conclude that Varun did not subscribe to a + b + g = 16 Choice (4) India-These Days. Choice (3) 16. Given a = 6 12. If Kiranmala did not subscribe for India-These Days, Hence g = 6 and b = 2 then Varun subscribed for India-Day to Day. He learnt an instrument on a + b + g i.e 14 days in all. Choice (1) Choice (3)

Solutions for question 13: 17. From the given pattern we can understand that it is a cyclic pattern. 13. Given out of the six baskets four baskets have either Hence the input is repeated in every 7th step. gold or silver. Therefore step 28 would be the same as the input Choice (4)

LA (Circular Arrangements)

1. The sitting arrangement was as follows

B B F/H A/H F A

H/A C H/F C

G D G D

E E Therefore from the above possibilities we can conclude that H is sitting opposite to C or D. Choice (4)

2. From option (a) U cannot see P, W and Q, so option (a) is not correct P T W From option (b) P W T

Q R QS

S U V R

V Y X Y U 21 X All the conditions are satisfied, so (b) can be the answer Hence we can say that the Mumbai co-ordinator is opposite the Delhi co-ordinator. Choice (1) From option (c) P Solutions for question 4: S W 4. Given R and T sit together. We can arrange them in 2 T Q ways. P and S do not sit together, so we can arrange them in 6 ways.

R V

U X Y T cannot see y, so option (c) is not correct From option (d) T/R P Q T R/T

W S S

S

V U

Y R

X P T/R P T/R

Bring the line below the diagram U cannot see P, so R/T R/T Choice (4) is not correct Choice (2)

Solutions for questions 3: P S

Using the first clue we can draw the figure as follows:

H T/R

R/T

We can arrange the remaining two employees Q & U in 2 ways. Therefore, the total number of ways = 2 × 6 × 2 M = 24 ways

B Alternate solution:

Considering R and T as a single unit, we get 3 units Using the 2nd clue we get, (R, T), Q and U which can be arranged around the table (3 – 1)! × 2! ways [2! since RT can be arranged H among the themselves] Now the remaining 2 persons 3 × D can be placed in 2 of the 3 positions in C2 2! ways. Therefore the total number of arrangements = (3 – 1)! 3 (2!) ( C2) 2! = 24 ways. Choice (B)

M

B LA (Distribution)

Solutions for questions 1 to 3:

From the given data we can conclude that the twelve persons were living in the building as follows.

Key board Guitarist Singer Instrumentalist Drummer Singer Drummer Singer Guitarist Guitarist Singer Instrumentalist player F S B E/B T G/E Q C R D P A Floor 1 Floor 2 Floor 3 Floor 4 Floor 5 Floor 6 Floor 7 Floor 8 Floor 9 Floor 10 Floor11 Floor 12

22 1. The four singers were S, T, C and P. Therefore only on November 15th, 1998.and the one who got married one male singer was there in the band. Choice (1) on January 3rd, 1999. And it was 49 days. Choice (1)

2. Above the floor in which G lived there were 6 floors or 8 6. Based on the conditions given, the groups are as floors. Choice (4) follows: Jalan, Kokila and Kadambar 3. S lived in the second floor. Choice (2) Jagan, Kavya and Kavita, Jeevan, Kavya and Kekul Therefore Jagan is in the same group as Kavya and Solutions for questions 4 and 5: Kavita Choice (1)

4. From the given data, the only possibility is 7. Given B went to college on Thursday and did not teach 1993 November 5th Physics. As A, B and C did not teach Physics, we can 1994  conclude that D taught Physics. 1995  As A and C went to college on consecutive days it can 1996  February 29th be either on Monday and Tuesday, or Tuesday and 1997  April 23rd, October 15th Wednesday. 1998  November 15th They cannot go to college on Tuesday and Wednesday 1999  January 3rd. as Physics is taught after Chemistry. Both the couples who got married in the same month Hence A and C go to college on Monday and Tuesday got married in the month of November. Choice (3) respectively. ∴ Biology is taught on Monday. 5. The least difference between the marriage dates of any Choice (1) two couples was between the couple who got married

Solutions for questions 8 and 9:

Given Team I scored maximum number of points, 364 is at the 15th place and Team A got 361 points. Given the sum of the points scored by teams at (13 + 14 + 15) is 1046. 364 + 361 + x = 1046 ⇒ x = 321 Hence Team A is in the 14th place. As team C got 218 points and is in the 10th place and Team O got 251 points. Hence Team O should be placed between 11 -13. Given the ascending order of teams according to their points is O J F. Hence Team O is in the 11th place, team J is in the 12th place and Team F is in the 13th place with 321 points. Now we know the points of the teams in the 10th and the 11th place. Hence the points of the team in the 12th place is 284 points (from 10 + 11 + 12 = 753). As Team N and team B got 108 and 165 points respectively less than team F. Team N got 213 points and Team B got 156 points. The clue 7 + 8 + 9 = 590 implies that we can calculate the points of the third team in the above group. [We know N is one of the teams in the group] Hence 213 + 182 + x = 590. ⇒ x = 195 points. Hence Team N is in the 9th place. The clue Team E got 18 points less than Team N, implies that team E is in the 8th place. From the clue 4 + 5 + 6 = 412, we know B is one of teams in the group. Hence 116 + 156 + x = 412 ⇒ x = 140 Hence Team B is in the 6th place. The clue Team H got 4 points more than Team M, implies that Team K is in the 5th place. From the clue 1 + 2 + 3 = 302, we get 96 + x + 4 + x = 302 ⇒ x = 101 Hence Team M got 101 points and is in the 2nd place. Therefore Team H got 105 points and is in the 3rd place. The final arrangement is as follows.

1st 2nd 3rd 4th 5th 6th 7th 8th 9th 10th 11th 12th 13th 14th 15th Team L M H D K B G E N C O J F A I Points 96 101 105 116 140 156 182 195 213 2182 251 284 321 361 364

8. The required difference is 284 – 140 = 144 orders Fanta and V orders the same dish. Hence we know Choice (2) that V does not wear the Green shirt. Therefore S wear the

th Green shirt and he orders Pepsi. 9. The position of team E is 8 . Choice (2) Let us tabulate the data. Solutions for questions 10 and 11: Shirt Drink Given P wears the Orange shirt and he orders Sprite and P Orange Sprit the person wearing the Green shirt orders Pepsi. Q Coke From the clues, we know R wears the Red shirt, U wears the R Red Blue shirt, T drinks Thumsup and Q drinks Coke. S Green Pepsi Hence R, Q, T and U neither wear Green nor order Pepsi. T Thums up So the Green shirt is worn either by S or V. But from the U Blue clue, the person wearing Green shirt is the person who V 23 Now from the last clue the person wearing the Violet shirt It is given that the total work required to complete work ordered Maaza.     3  5   7  36 Hence V ordered Maaza and is wearing the Violet shirt. in A2 = N  D + N  D = (ND) 4  6   8  24

10. S drinks Pepsi. Choice (2) This will be true for infinite values of N. Thus a unique value of N cannot be determined from 11. V Violet shirt, Maaza is the correct combination. the given information. Choice (4) Choice (1) Line + Bar graph Solutions for questions 12 and 13: Solutions for questions 1 to 4: From the first and the last clue we get the following arrangement. 1. The number of accidents caused because of two 28 O L wheelers is × 75000 = 21000. 100 SA SL The accident severity index for two wheelers is 40. i.e. for every 100 accidents, 40 persons are killed. From the third clue we know that the cricketer from India is So for 21000 accidents 210 × 40 = 8400 persons are at the extreme left end of the row and he plays for RCB. killed. Choice (3) Also P is to the immediate right of him who plays for DC.

P O L 2.

IND SA SL DC Total Persons Persons RCB x accidents killed injured nd Now from the 2 clue we know that N is to the immediate Trucks 19500 6630 12870 0.52 left to the player who plays for KKR. Bus 13500 4050 9450 0.43 From the above consideration we get the final arrangement as Car 12000 4200 7800 0.54

M P N O L Two 21000 8400 12600 0.67 AUS, ENG, SA wheelers IND, SL DC RCB MI KKR DD Others 9000 4050 4950 0.82

12. M plays for RCB and comes from India. x = the ratio of the persons killed to the persons injured. Choice (1) The required ratio is the highest for other types of accidents. Choice (2) 13. The player from Australia plays for DC. Choice (4) 3. The number of people who got injured by car accidents Solution for questions 14 and 15: was 7800 Choice (2)

14. Given X gave the presentation before T, R gave the 4. The number of persons killed in truck is 6630 and presentation before V but after U, also W gave the number of persons injured in other type of accidents is presentation after P and S but before U and X. 4950. So, Q, U, X, T, R and V (need not be in the same order) The required difference is 1680 Choice (4) conducted the seminars after W. Hence W should be giving his presentation in group 1 with P and S. DI (Distribution) Choice (1) Solutions for questions 1 and 2: 15. From the choices we can say that X, U and Q can be nd As policies mature in between 1997 and 2002 and a policy the first person to give the presentation in 2 group but th th matures on Feb 29 , the policy should mature on 29 Feb V cannot be the person. Choice (4) 2000. th Now the policy which matures on Jan 10 matured after 16. Given R attends the Physics tuition, Q attends the th th Feb 29 . Hence the policy on Jan 10 can be in 1997 or in Maths tuition. Also P and U same tuition, T and V 2002. attend same tuition and S does not attend the same st But from one of the clues the policy on May 21 is the last tuition as Q. th st and the Sep 17 policy is immediately before the May 21 Hence S should attend a tuition with at least policy. 2 students in it. i.e. S should either attends Physics or th th Hence the policies on Sep 17 and Jan 10 should mature Chemistry as there should be at least 2 students in the same year. each tuition. Q should attend a tuition to which th th The policy on August 8 is before Feb 29 . 3 people go. Choice (4) th Hence August 8 policy is for either 98 or 99.

The final arrangement is as follows. Quant ERPV 1st 2nd 3rd 4th 5th 6th th 1. The ratio of the floor areas of A1 and A2 is 1 : 4 th th 29 Feb th th st 24 Aug' 8 Aug 10 Jan 17 Sep May 21 Now the total work required to complete the work in 2000 1997 1998/1999 2001 2001 2002   1 N  5  9 A1 = N (D) +  D = ND, where D = no. of 6 4  6  24 th 1. The third matured policy is on 29 Feb 2000 hours Choice (1) ∴ The total work required to complete the work in A  9  2. In the year 2001, Atul receives money from two policies. must be 4  ND .  24  Choice (2) 24 LA(Linear Arrangement)

Solutions for questions 1 to 3:

Given S is in 3rd place from the left end and the positions of F, Q, G are also given.

S F Q G UK As two of F, Q, G are from USA and there is atleast one person between any two friends from USA, we can say that F and G are from USA. Also the extreme ends are occupied by friends from USA.

S F Q G USA UK USA USA USA USA

Now the friends from UK are separated by atleast four friends. So the other friend from UK can come either in the 2nd or the 4th position from the right. But from one of the clues P is from Australia and is in between Ι and J. Hence we can get it as.

Ι Ι S F Q G /J P J/ USA Aus UK USA Aus USA UK USA Aus USA

Now as R is adjacent to Ι and the friends at extreme ends are of different gender, we can get the final arrangement as follows.

T H S F Q G R Ι P J USA Aus UK USA Aus USA UK USA Aus USA

1. F, G, Ι and J stay in the USA. Therefore four female Now D got the 4th rank ⇒ D gets less than C. But the friends stay in the USA. Choice (4) condition that one pair should have same marks is not satisfied. Hence E should get the highest marks. 2. Three friends are in between the friends from the UK So E > C > A Choice (1) Now D cannot have more marks than C as D should have marks less than 3 students, hence C > D. 3. T is the only male friend from USA. Choice (2) As the first and the last ranked students do not have same marks as any other student, B should get the same marks Solution for question 4 and 5: as 'C'. Hence all the conditions are satisfied and the final Given from (i) and (iv), the arrangement would be arrangement is E > C = B> D > A.

S R 1. E got the highest marks. Choice (4)

2. B and C got the same marks. Choice (2) From the (iii) clue, U and V have 2 persons in between them. 3. The descending order is ECBDA. Choice (1) Hence we have only one possibility.[since there is only one person between Tarun and Qureshi] Solutions for questions 4 and 5:

S U/V R V/U Given at least 2 movies were released before R and there is one movie released between S and T. Hence R cannot be released on the 3rd or the 4th week of the From the (ii) clue we get the final arrangement as month. It can only be released on the last week. Therefore Q is released on the 4th week. As S and T have nd S P U/V R T V/U Q one movie released between them, P is released on the 2 week. Hence the final order would be as S/T, P, T/S, Q, R. 4. Raju is 3 places away to the left of Qureshi. Choice (4) 4. R is released last. Choice (3)

5. Pradip is to the immediate right of Shyam. 5. Only one movie is released before P. Choice (1) Choice (3) Line Graph + Table LA (Sequencing) Solutions for question 1: Solutions for 1 to 3: 1. From the table we can identify that the profit/ton in 2000 From the clue, D got less than 3 other students hence D got is the highest for Steel. From the line graph it is clear the 4th rank. that the production of Steel is the highest. From the other clue E > C > A. Hence the profit of Steel should be the highest. Let us assume B got the highest marks. Then the order is B Choice (4) > E > C > A.

25